Re: [obm-l] Curvas

2003-07-21 Por tôpico Alexandre Tessarollo

Ambas as curvas são hipociclóides. Como disse o Dirichlet, são obtidas através do 
"rastro" de um ponto fixo numa circunferência pequena rodando dentro de uma maior. Só 
para não deixar margens a dúvidas, vale ressaltar que são circunferências tangetes 
internamente.

A figura 1 é um hipociclóide tricúspide ou, se vc preferir, um deltóide. Possui 
diversas propriedades interessantes - por exemplo, a envoltória das tangentes é um 
outro deltóide. Tb está intimamente ligado ao desenho da elipse.

Já a figura 2 especificamente eu não conheço. Se tivesse que chutar diria que não 
possui nenhum nome exclusivo. Contudo, certamente tem algum nome "genérico" e 
propriedades "genéricas".

Se vc realmente estiver interesado, posso depois vasculhar meus alfarrábios para te 
dar maiores detalhes de como construir, propriedades, equações, curvas relacionadas, 
etc. Só pedir que eu venço minha preguiça :)

[]'s

Alexandre Tessarollo
-- 
__
Sign-up for your own FREE Personalized E-mail at Mail.com
http://www.mail.com/?sr=signup

CareerBuilder.com has over 400,000 jobs. Be smarter about your job search
http://corp.mail.com/careers

=
Instruções para entrar na lista, sair da lista e usar a lista em
http://www.mat.puc-rio.br/~nicolau/olimp/obm-l.html
=


Re: [obm-l] Como os Matemáticos Complicam II

2003-07-11 Por tôpico Alexandre Tessarollo
Meu caro João Paulo,

Pelo que entendi você não quer exatamente a resolução de um problema quando o coloca 
aqui - vc quer sim saber de onde diabos aquilo surgiu e por que razão vc deveria 
resolvê-lo.

Aviso logo que este é um e-mail longo. Desde já minhas desculpas a todos e em 
particualr ao N pelo off-topic (nem tão off assim) - espero que esse meu e-mail ajude 
a esclarecer as coisas e a encerrar essa discussão.

**

>Matemática,pra mim não tem a ver com o lado emocional.Só não consigo
entender bem uma coisa que não estou vendo um motivo lógico pra
existir(Caso de raiz,x e y etc)

Vc vê motivo lógico para os poemas de Vinícius de Moraes existirem? Assim como os 
poemas, raízes são desdobramentos e evoluções de determinados campos do conhecimentos, 
de determinadas formas de expressão. Tanto a matemática como a poesia são formas do 
ser humano se expressar e são campos de conhecimento, em constante evolução. Às vezes 
é difícil entender como surgiu um Vinícius de Moraes, mas se considerar toda a base 
literária e musical que veio antes, um Vinícius torna-se natural.

Assim com os poetas, assim com a matemática. Talvez o que esteja lhe faltando seja uma 
boa aula de história da matemática. Aí vc vai entender o que veio de one. Desde o 
pastor contando suas ovelhas no pasto até os nomes esquisitos que vc vê pro aqui, como 
integrais e equações diferenciais.

Ah sim, só para deixar um teaser: E qual o motivo lógico para existir Astronomia? De 
que me serve saber que a nebulosa de sei-lá-o-que é composta disso e daquilo outro? E 
para que Música? Qual a utilidade de se tocar um instrumento ou ficar criando novos 
instrumentos?

**

>É muito mais fácil eu entender um texto todo em japonês.Pelo menos
posso ir atrás de um dicionário e traduzir.
>Já com matemática,não tem aonde ir.

Tem sim. Livros de matemática, professores, colegas que saibam um pouco mais que você 
e fóruns como esse. Contudo, é sempre mais fácil se você usar abordagens mais sutis. 
Assim como você não pede uma informação na rua dizendo mil palavrões e gritando a 
altos brados que aquele bairro não deveria existir, tb com matemáticos é mais 
apropriado um tanto de gentileza. Afinal, somos todos humanos.

Eu mesmo já tive vários alunos que me disseram que não gostavam de matemática. Nunca 
tive problemas com isso. É como dizer ao fã de uma banda que vc odeia aquele grupo. Se 
os dois conseguirem se expressar claramente e fundamentar seus pontos de vista, vcs 
certamente não vão brigar. Podem não mudar de opinião, mas vcs vão entender pq o outro 
gosta/não gosta tanto assim da banda.

Outra coisa que ajuda muito é aprender o básico da língua da pessoa. Se vc chega para 
um londrino e pergunta "onde á a pizzaria mais próxima" ele não vai entender lhufas. 
Já se vc disser "Pizza! Where? Where pizza?" ele, com um tanto de boa vontade, vai 
entender e te apontar algo como "one two three right then one two left" para dizer que 
é a terceira quadra à direita e então a segunda à esquerda. Sei que o exemplo parece 
bobo, mas acho eu que ilustra bem.

**

Já expûs como descobrir de onde vieram os problemas e conceitos matemáticos (curso de 
história da matemática) e o porquê da dificuldade de conseguir ajuda em matemática 
(abordagens mais gentis e predisposição favorável sempre ajudam a se conseguir o que 
se quer de um humano). Falta dizer agora por que diabos vc deveria resolver probs de 
matemática.

Bem, afora os motivos mais óbvios e terrenos - passar na escola, no vest, etc - 
existem alguns outros:

1) Desenvolver e treinar o raciocínio lógico

Resolver problemas de matemática é como tentar pintar um desenho respeitando as 
bordas. Achar que x=4 ou ter um desenho pintado não serve de muita coisa, mas saber 
observar um problema, saber dizer quais são os dados, saber escolher a melhor 
abordagem, conseguir implemntar essa abordagem e achar a resposta são coisas que 
treinam o grande "músculo" que é nosso cérebro. Ficamos com um raciocínio lógico 
melhor - que nos permitirá depois calcular que não vai dar para chegar no outro bairro 
em menos de dez minutos *e* conseguir entradas para o cinema *portanto* vamos naquela 
outra sala mais perto. Mesma coisa para a aula de artes - ninguém tem um diploma de 
que conclui o curso de respeitar as bordas com louvor, mas é algo que treina sua 
coordenação motora fina e sua capacidade de concentração e estimula sua acuidade 
visual.

2) Resolver problemas práticos

Se vc vai preparar um receita e na hora vê que só tem metade de determinado 
ingrediente, vc precisa saber que só poderá usar a metade de todos os outros 
ingredientes para manter a proporcionalidade.

Já se vc pega um receita que dá para 8 pessoas e quer preparar para 20, vc precisa ter 
noção de que usará duas vezes e meia todos os ingredientes.

Mesmo um professor de português precisa saber calcular suas médias - e nem sempre é 
tão fácil. Muitas escolas trabalham com um sistema de pesos diferentes para cada nota 
que, quando chega no final do

Re: [obm-l] POLEMICA

2003-07-11 Por tôpico Alexandre Tessarollo
Operação 1:
Compra por 20 e vende por 30. Até aqui lucro líquido de 10. O sujeito tira 20 do bolso 
e ganha 10 extras - nesses termos são 50% de lucro sobre o preço de compra.

Operação 2:
Recompra por 50 e revende por 60. Aqui ele entra com todo o anterior (20 iniciais mais 
10 de lucro inicial) mais 20 do próprio bolso e sai com 60, sendo (20 iniciais) + (10 
de lucro inicial) + (outros 20 do bolso) + (10 de lucro novo). Nessa operação o lucro 
líquido é de 10. Se vc consicerar como investimento apenas os 20 que ele tira do 
bolso, o lucro aqui foi de 50%.

O investimento total do cara (o que saiu do bolso dele) foi 40. Ele lucrou (10 
iniciais) + (10 na segunda operação). Portanto, teve lucro de 50% com base no 
investimento total.

Uma outra maneira de pensar o problema seria ver que ele investiu (investir=tirar do 
próprio bolso) 40 e no final recebeu 60 de crédito. Portanto, 60-40=20 de lucro. 
Novamente, 50% sobre o investimento total.


[]'s

Alexandre Tessarollo
-- 
__
Sign-up for your own FREE Personalized E-mail at Mail.com
http://www.mail.com/?sr=signup

CareerBuilder.com has over 400,000 jobs. Be smarter about your job search
http://corp.mail.com/careers

=
Instruções para entrar na lista, sair da lista e usar a lista em
http://www.mat.puc-rio.br/~nicolau/olimp/obm-l.html
=


[obm-l] Re: PAs de ordens>1

2002-11-22 Por tôpico Alexandre Tessarollo

   Antes de mais nada, obrigado pelas respostas do N para os raios e de todos que 
responderam às questões do somatório de x^2 e da PA de k-ésima ordem. Gostaria de 
comentar a resposta do Domingos Jr. em particular:

>>   Ou, mais genericamente, como se calcula a soma do n primeiros termos de uma PA de 
>2a >>ordem, onde b[n+1]-b[n]=a[n], sendo a[n] o termo de uma PA "normal"(de 1a 
>ordem)? >>Naturalmente temos a[1], R e b[1].
>
>o somatório de 1 até n dos a[i] vai dar: n.a[1] + somatório{i = 1 até n-1}[(n-i).b[i]]
>
>a1 = a1
>a2 = a1 + b1
>a3 = a1 + b1 + b2
>...
>an = a1 + b1 + b2 + ... + b[n-1]
>a1 + a2 + ... + an = n.a1 + (n-1).b1 + (n-2)b2 + ... + b[n-1]

   Ok, acho q vc assumiu que b[i] era a PA de primeira ordem e a[i] seria a PA de 
segunda ordem. Considerando assim, está perfeitamente corrto o que vc escreveu. Nos 
meus rascunhos eu tinha chegado exatamente até esse ponto. O problema que eu tive foi 
expressar a soma da PA de 2a ordem em função APENAS de a[1], b[1], n e R (razão da PA 
d 1a ordem).

   Concordo que a sua resposta faz isso IMplicitamente, mas eu gostaria de algo 
EXplícito. 

   Por analogia, poderíamos dizer que o somatório da PA de 1a ordem de b[1] até b[n] 
pode ser escrito como S[n]=(b[1]+b[n])n/2, mas eu prefiro expressar explicitamente, ou 
seja, S[n]=(2b[1]+(n-1)R)n/2.

   Desculpem a falta de clareza anterior. Ah sim, ainda falta alguém se manifestar 
quanto a generalização dessa pergunta, ou seja, como expressar o somatório de uma PA 
de k-ésima ordem em função (explícita :-)) dos primeiros termos de cada PA de ordem 
inferior?

[]'s

Alexandre Tessarollo
-- 
__
Sign-up for your own FREE Personalized E-mail at Mail.com
http://www.mail.com/?sr=signup

One click access to the Top Search Engines
http://www.exactsearchbar.com/mailcom

=
Instruções para entrar na lista, sair da lista e usar a lista em
http://www.mat.puc-rio.br/~nicolau/olimp/obm-l.html
O administrador desta lista é <[EMAIL PROTECTED]>
=



[obm-l] PAs de ordens>1

2002-11-21 Por tôpico Alexandre Tessarollo

   Estou num momento de diarréia mental. Qual é e como deduzir a fórmula de somatório 
de x^2, para x=1,2,..,n? 

   Ou, mais genericamente, como se calcula a soma do n primeiros termos de uma PA de 
2a ordem, onde b[n+1]-b[n]=a[n], sendo a[n] o termo de uma PA "normal"(de 1a ordem)? 
Naturalmente temos a[1], R e b[1].

   Generalizando ainda mais, sejam a{1}[1], a{2}[1],..,a{k}[1] respectivamente os 
primeiros termos de PAs de 1a, 2a,..,k-ésima ordem e R a razão da PA de primeira 
ordem. Em função desses parâmetros, qual a soma dos n primeiros termos da PA de 
k-ésima ordem?

[]'s

Alexandre Tessarollo
-- 
__
Sign-up for your own FREE Personalized E-mail at Mail.com
http://www.mail.com/?sr=signup

One click access to the Top Search Engines
http://www.exactsearchbar.com/mailcom

=
Instruções para entrar na lista, sair da lista e usar a lista em
http://www.mat.puc-rio.br/~nicolau/olimp/obm-l.html
O administrador desta lista é <[EMAIL PROTECTED]>
=



[obm-l] Raios num triângulo qualquer

2002-11-21 Por tôpico Alexandre Tessarollo

   Como posso calcular o raio da circunferência inscrita de um triângulo qualquer em 
funçào dos lados? E da circunferência circunscrita?

   Comecei a divagar em cima disso pensando no problema abaixo:
 "Seja I o incentro do triângulo ABC. Dados que AB.BC=AC^2-AB^2 e AI=BC-AC, prove que 
AB^2/(AI.BI)+BI/AB=(BI/AI)^2"

   Alguém se habilita a me dar uma ajuda? Plz..

[]'s

Alexandre Tessarollo
-- 
__
Sign-up for your own FREE Personalized E-mail at Mail.com
http://www.mail.com/?sr=signup

One click access to the Top Search Engines
http://www.exactsearchbar.com/mailcom

=
Instruções para entrar na lista, sair da lista e usar a lista em
http://www.mat.puc-rio.br/~nicolau/olimp/obm-l.html
O administrador desta lista é <[EMAIL PROTECTED]>
=



[obm-l] Re: exponencial

2002-11-04 Por tôpico Alexandre Tessarollo
>Resolva:
>[( raiz quadrada de 3) + 1]^x + [( raiz quadrada de 3) - 1]^x = 8
>
>
>Graficamente vejo duas soluções: uma positiva ( x = 2)e outra
>negativa.
>Me pediram algebricamente. Divido com vocês a dor de cabeça.

=+=+=+=+=+=+=+=+=+=+=+=+=+=+=+=+=+=+=+=+=+=+=+=+=+=+=+=+=+=+


 Nooossa, quanto tempo faz que não escrevo aqui... Bem, de qq forma, vamos lá, de cara 
partindo p/a grosseria: Vou usar o Binômio de Newton, que diz que
(x+a)^n=sum[p=0, n]{Cn,p*(a)^p}*(x)^(n-p))
Leia-se "... igual a somatório, de p=0 até p=n, de ..."

 No nosso caso, vou expandir (sqrt3+1)^x + (sqrt3-1)^x de uma só vez. Fica:
sum[p=0, x]{Cx,p*(1)^p*(sqrt3)^(x-p)} + sum[p=0, x]{Cx,p*(-1)^p*(sqrt3)^(x-p)}

Juntando num mesmo somatório e colocando a combinação e o sqrt3 em evidência
sum[p=0, x]{Cx,p*(sqrt3)^(x-p)((1)^p+(-1)^p)}

Devido ao (1)^p+(-1)^p, podemos afirmar que o somatório só vai considerar as parcelas 
com p=2k. Substituindo p por 2k e admitindo x PAR, temos:
sum[k=0, x/2]{Cx,2k*(sqrt3)^(x-2k)*(2)} =
= 2*sum[k=0, x/2]{Cx,2k*(sqrt3)^(x-2k)}

 Se x for ímpar, o somatório vai de k=0 até k=(x-1)/2. Até aqui eu acho que está tudo 
certo. Agora pretendo calcular os somatórios em separados, isto é, calcular o sum de 
Cx,2k e depois o de (sqrt3)^(x-2k).

Primeiro, o de Cx,2k. Montando o triângulo de Pascal, temos

1
1 1
1 2 1
1 3 3 1
1 4 6 4 1
etc

Vale lembrar que o somatório de Cx,p é a soma da linha e que, o número i da linha é 
dado por i=x+1. (Lembra que x era o expoente dos nossos binômios?)
Eliminando os termos da forma Cx,2k+1, temos

1
1 
1 1
1 3 
1 6 1

Creio que seja fácil ver que a soma da linha i é 2^(i-2).Ou, em outras palavras, digo, 
variáveis, o somatório de Cx,2k vale 2^(x-1).
 
Agora, calculemos o de (sqrt3)^(x-2k). Se vc substituir os valores iniciais de k, verá 
que se trata de uma PG. Calculando a soma dessa PG num rascunho, achei (3^(x/2+1)-1)/2.

Tomando o somatório que tínhamos e substituindo pelos valores encontrados
2*sum[k=0, x/2]{Cx,2k*(sqrt3)^(x-2k)}=
= 2*sum[k=0, x/2]{Cx,2k}*sum[k=0, x/2]{(sqrt3)^(x-2k)}=
= 2*2^(x-1)*(3^(x/2+1)-1)/2 =
= 2^(x-1)*(3^(x/2+1)-1)

Bem, lembrando nós só trabalhamos o lado esquerdo da igualdade proposta pelo problema, 
podemos igualar o nosso resultado ao lado direito da igualdade. Teremos:

2^(x-1)*(3^(x/2+1)-1)=8 PARA X PAR (lá no meio dos somatórios eu supus que x era par, 
lembra?)

Para x ímpar, basta fazermos algumas alterações. Se não errei nada, fica
2^(x-1)*sqrt3*(sqrt3^(x+1)-1)=8


 E é no momento em que lembramos que x=2 era raiz que eu lembro porque tinha parado de 
escrever e-mails na madruga :-) Se vc fizer x=2 na equaçào de x par, vc acha que 
16=8!! Ou eu acabei de revolucionar completamente a matemática ou, o que é mais 
provável, cometi algum erro crasso nas minhas contas. Já revisei-as umas 3 vezes, mas 
não consegui detectá-lo. Alguém se habilita?

[]'s

Alexandre Tessarollo
-- 
__
Sign-up for your own FREE Personalized E-mail at Mail.com
http://www.mail.com/?sr=signup

=
Instruções para entrar na lista, sair da lista e usar a lista em
http://www.mat.puc-rio.br/~nicolau/olimp/obm-l.html
O administrador desta lista é <[EMAIL PROTECTED]>
=



[obm-l] Re: Uma equaçao...

2002-10-07 Por tôpico Alexandre Tessarollo


   Só p/convencionar, log[b]{a} é "log de a na base b", ok? Assim, seu prob fica:

log[2x]{2x^(-1)}*(log[2]{x})^2+(log[2]{x})^4=1

Como log[b]{a^n}=n*log[b]{a}, podemos mexer no primeiro fator. Fica:
(-1)*log[2x]{2x}*(log[2]{x})^2+(log[2]{x})^4=1

Como log[a]{a}=1, podemos melhorar ainda mias o primeiro fator. Teremos:
(-1)*(1)*(log[2]{x})^2+(log[2]{x})^4=1

Chamndo log[2]{x}=L, temos

-L^2+L^4=1
L^4-L^2-1=0

Resolvendo a biquadrada (imagine L^2=y. Resolva a eq do segundo grau em y. Iguale L^2 
às duas raízes. Uma não presta pq é negativa, e da outra, temos 
L=+-sqrt((1+sqrt(5))/2).

Teremos log[2]{x}=+-sqrt((1+sqrt(5))/2)
Sabemos que, se log[b]{a}=c, então b^c=a. Fazendo isso, temos
x=2^sqrt((1+sqrt(5))/2)
ou
x=2^-sqrt((1+sqrt(5))/2)=1/(2^sqrt((1+sqrt(5))/2))

[]'s

Alexandre Tessarollo

PS: Vale lembrar q sqrt(x)="raiz quadrada de x". Qq dúvida, na notação ou na 
resolução, escreva. 
-- 
__
Sign-up for your own FREE Personalized E-mail at Mail.com
http://www.mail.com/?sr=signup

"Free price comparison tool gives you the best prices and cash back!"
http://www.bestbuyfinder.com/download.htm

=
Instruções para entrar na lista, sair da lista e usar a lista em
http://www.mat.puc-rio.br/~nicolau/olimp/obm-l.html
O administrador desta lista é <[EMAIL PROTECTED]>
=



[obm-l] Re: dificuldade

2002-08-18 Por tôpico Alexandre Tessarollo


   Admitindo que a ordem em cada banco importa, isto é, existe o lugar à direita e o à 
esquerda no banco, basta fazermos uma permutação circular com todas as 24 pessoas. A 
fórmula para permutação circular de n é (n-1)!, logo, existem 
23!=25.852.016.738.884.976.640.000 maneiras desse povo subir na roda gigante.

   Já se vc estiver se importando apenas com que dupla vai se sentar aonde, ou seja, 
tanto faz quem vai sentar de um lado ou de outro, temos 2 maneiras: A primeira é 
dividr o resultado anterior por 2^12. (Vale lembrar que uma vez escolhida a dupla, 
existem 2 maneiras desses dois se sentarem AB e BA). Portanto, 
23!/2^12=6.311.527.524.141.840.000 maneiras.

   A segunda maneira seria separar primeiro as duplas 
[C(12,2)*C(10,2)*C(8,2)*C(6,2)*C(4,2)*C(2,2)] e depois fazer a permutação circular de 
12 duplas [11!]. Multiplica um número pelo outro e vc terá o mesmo resultado de antes.

[]'s

Alexandre Tessarollo
-- 
__
Sign-up for your own FREE Personalized E-mail at Mail.com
http://www.mail.com/?sr=signup

=
Instruções para entrar na lista, sair da lista e usar a lista em
http://www.mat.puc-rio.br/~nicolau/olimp/obm-l.html
O administrador desta lista é <[EMAIL PROTECTED]>
=



[obm-l] Re: mais que CRUEL

2002-08-16 Por tôpico Alexandre Tessarollo


> 
> Date: Fri, 16 Aug 2002 13:28:59 +
> From: "Paulo Santa Rita" <[EMAIL PROTECTED]>
> Subject: [obm-l] mais que CRUEL
> 
> Ola Alexandre e demais
> colegas desta lista ... OBM-L
> 
> Talvez nao seja exagero dizer que o problema a que se refere a mensagem do 
> Alexandre, abaixo, é mais que que CRUEL ...
> 
> Um dos discipulos de EULER propos ao seu mestre o seguinte problema :
> 
> Num poligono convexo de N lados, no qual duas diagonais quaisquer não sao 
> paralelas, quantos pontos no exterior do poligono sao pontos de interseccoes 
> de diagonais se, nesta regiao exterior e tambem no interior do poligono, 
> nenhum ponto e intersecao de mais de duas diagonais ?
> 
> EULER nao resolveu a questao, nao obstante ter lutado durante cerca de tres 
> meses com ela. O discipulo resolveu, alguns meses apos, mas morreu muito 
> jovem e nao se tornou o Grande Matematico que todos esperavam.
> 
> Eu nao li esta historia. A pessoa que me propos a questao contou. Independe 
> de tudo isso, a existencia da historia e o fato do problema ser antigo sao 
> indicativos de sua complexidade.
> 
> A ideia que tive e muito parecida com a do Tessarolo, na mensagem abaixo.
> 
> Existem D=(N(N-3))/2 diagonais. Duas diagonais quaisquer representam um 
> ponto, que e a interseccao entre elas. Claramente que em um vertice mais que 
> uma combinacao esta representando o mesmo ponto ...
> 
> As intersecoes podem ser :
> 
> 1) Um vertice ( total S1)
> 2) Num ponto no interior do poligono (total S2)
> 3) Num ponto no exterior do poligono (total S2)
> 
> Claramente que :
> 
> S1+S2+S3 = total de interseccoes !
> 
> Num vertice concorrem N-3 diagonais, isto fornece BINOM(N-3,2). Ha, porem, N 
> vertices. Segue que :
> 
> S2 + S3 = BINOM(D,2) - N*BINOM(N-3,2)
> 
> O numero  BINOM(D,2) - N*BINOM(N-3,2) e a soma dos pontos de interseccoes no 
> interior e no exterior do poligono.
> 
> Aqui, agora, entra a ideia do Tessarolo :
> 
> Uma diagonal e uma cisao nos vertices. Se { V1, V2, ..., Vn} sao os vertices 
> do poligono, a diagonal {V1,V5} cinde os vertices em dois subconjuntos : 
> {V2,V3,V4} e {V6,V7,...,Vn}. Escolhendo um ponto qualquer no primeiro 
> conjunto e um ponto qualquer no segundo, teremos uma interseccao com a 
> diagonal {V1,V3}. O total de escolhas possiveis e uma mera aplicacao do 
> principio multiplicativo da analise combinatoria e entao passa-se ao 
> somatorio.
> 
> Se o poligono tem N lados, basta se computar as diagonais que tem ate [N/2] 
> lados do poligono, onde [N/2] e o maior inteiro que nao supera [N/2].
> 
> Exemplo: Poligono convexo de 9 lados
> 
> Vertices : {V1,V2,...,V9}
> 
> Diagonais : {V1,V3},{V2,V4},{V3,V5}, ..., {V9,V2}
> Estas diagonais tem, ao seu lado, 2 lados do poligono
> 
> Diagonais : {V1,V4},{V2,V5},{V3,V6},...,{V9,V3}
> Estas diagonsi tem, ao seu lado, 3 lados do poligono
> 
> Diagonais : {V1,V5},{V2,V6},{V3,V7},...,{V9,V4}
> Estas diagonais tem, ao seu lado, 4 lados do poligono
> 
> como 4=[9/2]. Paramos aqui !
> 
> A diagonal {V1,V6} ja foi computada.
> 
> O calculo das somas acima e facil. O somatorio fornecera S2. Dai, usando
> S2 + S3 = BINOM(D,2) - N*BINOM(N-3,2), calculamos S3. E o tio Euler descansa 
> em paz !
> 
> Todavia, tudo isso e burocracia e malabarismo. A ideia brilhante e genial 
> cabe ao Tessarolo, com sua visao de cindir os vertices em dois conjuntos. E 
> sem saber ele resolveu um problema que venceu o Tio Euler...
> 
> Claramente que fizemos as restricoes necessarias. Uma abordagem da 
> existencia de diagonais paralelas talvez fique melhor no ambiente da 
> Geometri Analitica, mais que no da sintetica.
> 
> Seja da um poligono convexo de N lados, com vertices 
> (X1,Y1),(X2,Y2),...(Xn,Yn). Que condicao(oes) devem satisfazer estes pontos 
> para que existam ao menos duas diagonais paralelas ? É possivel caracterizar 
> todos os feixes de paralelas que podem existir ?
> 
> Um abraco a todos
> Paulo Santa Rita
> 6,1028,160802


   Muito obrigado pelos elogios. Realmente são de levantar a moral de qualquer um :-))

   Mas vamos ao que interessa: Por um certo apreço a pouca sanidade que me resta, 
ative-me aos polígonos regulares. Acreditei que suas particularidades deveriam nos 
ajudar um pouco.

   Como disse o Paulo, existem 3 tipos de intersecções. Vou acrescentar um quarto 
tipo, "entre diagonais paralelas". (Só p/facilitar as contas. Contudo, se vc 
acrscentar completar o plano euclidiano com a linha no infinito... :-)))
   
1) Um vertice ( total S1)

   Dada uma diagonal qualquer do polígono, ela intersecta outras 2(n-4) diagonais em 
um dos seus 2 vértices. Tomemos a diagonal V1V5. O total de diagonais que possuem V1 
OU V5 com vértice (afora a própria V1V5, claro) é sempre (n-4)+(n-4).

   Logo, S1=2(n-4)

2) Num ponto no interior do poligono (total S2)

   Bem, se o que falei naquele e-mail anterior estiver certo, então S2=somatório

3) Num ponto no exterior do poligono (total S3)

   Hum Vamos calcular S4 primeiro.

4) Diagonais paralelas (total S4)

 

[obm-l] Re: CRUEL

2002-08-15 Por tôpico Alexandre Tessarollo



   Estive fazendo umas contas e creio que posso ter chegado a uma resposta parcial. 
Por uma questão de praticidade, numerei os vértices de A[0] até A[n-1], no sentido 
anti-horário.

   Tome o vértice A[0] e vá ligando com os outros vértices (A[1]; A[2]; ...; A[n-1]). 
Note que cada diagonal A[0]A[i] divide o plano em dois semi-planos. Se o polígono é 
convexo, podemos determinar exatamente quantos vértices ficam em cada semi-plano.

   Para quem está com a figura, é fácil ver que, dada a diagonal A[0]A[i], todos os 
pontos A[j] com 0http://www.mail.com/?sr=signup

=
Instruções para entrar na lista, sair da lista e usar a lista em
http://www.mat.puc-rio.br/~nicolau/olimp/obm-l.html
O administrador desta lista é <[EMAIL PROTECTED]>
=



[obm-l] Re: Maio01

2002-05-13 Por tôpico Alexandre Tessarollo

Ok, mil desclupas. As palavras e os números se embolaram na minha frente... Agora 
lendo o enunciado com a devida 
calma, temos, no sentido anti-horário: AB=45; BP=45; PC=20; CD=20; DM=30; MA=30. 

Trace PD. Observe que o triângulo PCD é isósceles. Como o angPCD+angABP=180º, 
temos cos(PCD)=-cos(ABP). 
Projetando C ortogonalmente em AB, temos um triângulo retângulo do qula tiramos 
cos(ABP)=5/13, logo, cos(PCD)=-5/13. 
Aplicando Lei do Cossenos no triângulo PCD, temos PD=120/sqrt(5).

Aplicando Lei dos Senos em PCD, temos CP/sen(PDC)=PD/sen(PCD). Daí tiramos que 
sen(PDC)=sqrt(5)/5. Como 
angPDC+angPDM=90º, temos cos(ODM)=sen(PDC)=sqrt(5)/5. Aplicando Lei dos Cossenos em 
PDM, teremos 
PM=6sqrt(145).

Se eu não errei nenhuma conta no caminho, o resultado é esse. Outro opção seria 
tomaro o triângulo isósceles ABP ao 
invés do PCD. Depois, em vez de observar PDM, trabalha-se com PAM, mas o raciocínio 
todo é basicamente o mesmo.

[]'s

Alexandre Tessarollo

PS: Obrigado ao Antônio pelo sen(18). Já tinha ouvido o valor várias vezes mas nunca 
tinha parado p/pensar a respeito nem 
nunca tinha visto nenhuma demonstração. Valeu mesmo.




>Desculpe, mas o problema não fornece figura alguma, eu o passei como me foi 
>fornecido. A única coisa que se sabe sobre o 
>ponto P é que ele se localiza em BC, tal que BP meça 45, por conseguinte PC 20.

 - Original Message -
 From: "Alexandre Tessarollo" <[EMAIL PROTECTED]>
 To: <[EMAIL PROTECTED]>
 Sent: Saturday, May 11, 2002 5:28 AM
 Subject: [obm-l] Re: Maio01


 > > 2-No trapézio ABCD, o lado DA é perpendicular às bases AB e CD. A base AB mede 45, 
 >a base CD mede
 > > 20 e o lado BC mede 65. Seja P no lado BC tal que BP mede 45 e seja M o ponto 
 >médio de DA.
 > >
 > > Calcule a medida do segmento PM.

-- 
___
Sign-up for your own FREE Personalized E-mail at Mail.com
http://www.mail.com/?sr=signup

=
Instruções para entrar na lista, sair da lista e usar a lista em
http://www.mat.puc-rio.br/~nicolau/olimp/obm-l.html
O administrador desta lista é <[EMAIL PROTECTED]>
=



[obm-l] Re: Maio01

2002-05-11 Por tôpico Alexandre Tessarollo

> Pessoal vocês poderiam me ajudar nessas duas questões da olimpíada de maio de 2001 ? 
> 1-Em volta de um círculo situam-se dez moedas de 1 cm de raio . Cada moeda é 
>tangente ao círculo e
> às duas moedas vizinhas. Demonstre que a soma das áreas das dez moedas é o dobro da 
>área do
> círculo. 
>

   Junte os centros das moedas e vc terá um decágono regular de lado 2cm (2r para 
generalizar). Junte os ponto de tangência das moedas com o círculo e vc terá outro 
decágono regular, de lado L e inscrito numa circunferência de raio R. Observe o 
trapézio formado pelos centros (O1 e O2) de duas moedas consecutivas e seus 
respectivos pontos de tangência com o círculo (T1 e T2). Temos o trapézio O1T1T2O2. 
Sabemos que o ângulo interno de um decágono regular é 144º. (Odeio essa frase, mas) é 
fácil ver que T1O1 e T2O2 bissectam os ângulos O1 e O2 do decágono maior. Logo, NO 
TRAPÉZIO, os ângulos O1 e O2 são de 72º. A base maior O1O2 é 2cm (2r).

   Projetando T1T2 em O1O2 e usando cosseno nos dois triângulos retângulos que 
"sobram", vemos que T1T2 é igual à 2-2cos(72) [2r-2cos(72), na nossa generalização]. 
Assim nós temos o valor do lado do decágono menor. Pelas propriedades de polígonos 
regulares, sabemos que L/2R=sen(180/n), onde R é o raio da circunscrita e n é o número 
de lados. No decágono menor, L=2-2cos(72) [2r-2cos(72)]. Aplicando essa fórmula e 
fazendo as contas, temos R=(r-sen(18))/sen(18).

   Queremos provar que 2*Pi*R^2=10*Pi*r^2. Ou seja, queremos mostrar que R^2=5*r^2. 
Fazendo as devidas substituições e simplificando, só precisaremos saber o seno de 18º. 
Só que esse vale (sqrt(5)-1)/4. Simplificando devidamente, veremos que, para r=1, a 
afirmação é válida, CQD.

  Talvez eu não tenha sido claro o suficiente em alguns pontos pq estou com sono 
agora, mas os resultados que eu não msotrei são facilmente comprováveis - vide a 
relação eentre o lado do polígono regular e o raio da circunscrita. O único resultado 
que eu usei e não sei mostrar é o valor do sen(18). Esse eu realmente "colei"  :)))

> 2-No trapézio ABCD, o lado DA é perpendicular às bases AB e CD. A base AB mede 45, a 
>base CD mede
> 20 e o lado BC mede 65. Seja P no lado BC tal que BP mede 45 e seja M o ponto médio 
>de DA. 
>
> Calcule a medida do segmento PM. 
>

   Tá, DA=65. Mas cadê o ponto P? Meu e-mail atual é meio esquisitão, então nem sei se 
vc mandou um attach. Dei uma olhada no arquivo da lista e lá não tem anexo nenhum... 
Será q vc poderia descrever a posição de P?
> Obrigado 
>
> Marcus Dimitri 

[]'s

Alexandre Tessarollo
-- 
___
Sign-up for your own FREE Personalized E-mail at Mail.com
http://www.mail.com/?sr=signup

=
Instruções para entrar na lista, sair da lista e usar a lista em
http://www.mat.puc-rio.br/~nicolau/olimp/obm-l.html
O administrador desta lista é <[EMAIL PROTECTED]>
=



Re: [obm-l] Re: ???

2002-05-01 Por tôpico Alexandre Tessarollo



 
    Discordo da sua resposta... Observe o trecho do
enunciado: "...o  caminhão descarregou as mudas no canteiro
central,no local onde seria plantada a primeira muda... partindo do lugar
onde as mudas foram colocadas,ele [o jardineiro] pegou 3 mudas de cada
vez,plantou-as nos locais designados,enfileirando-as uma após a
outra..." Portanto, as mudas estvam no lugar da primeira e o jardineiro
parte necessariamente dali.
    O seu raciocínio me lembra um problema que
o Gugu e o Nicolau criaram a partir de uma situação real.
Se não me engano, eles estariam em Berlim, numa rua principal querendo
chegar em uma determinada transversal, percorrendo a menor distância
possível. Como não sabiam p/q lado ficava a transversal,
caminharam um quarteirão p/a direita, voltaram, foram um p/a esquerda,
voltaram, 2 p/a dir, volta, etc Assim, admitindo que os dois não
pudessem se separar, eles percorreriam a menor distância possível
até achar a rua.
    Fazendo um paralelo, o seu jardinerio estaria percorrendo
a menor distância possível p/plantar as mudas. Jardineiro
esperto esse, hein? :-))
 
[]'s
Alexandre Tessarollo
PS: N e Gugu, desculpem se deturpei a história. Creio q vcs usaram
a questão em alguma olimpíada e depois publicaram numa Eureka
contando a historinha... Mas isso diz a minha [péssima] memória
:-)
 
Eder wrote:
 Alguns colegas
meus acharam 67.Ninguém achou 65,ainda.Eu considerei que o jardineiro
partia do centro da fila de mudas e que ia colocando 3 mudas no lado direito,3
no lado esquerdo,...,e cheguei a uma distância total de 35 hectômetros.Porém,eu
somei a distância da última viagem,o que não seria
necessário,pela sua resolução.Vamos ver se alguém
esclarece isso...  - Original Message -

From:Alexandre
Tessarollo

To: [EMAIL PROTECTED]

Sent: Wednesday, May 01, 2002 4:26
AM

Subject: [obm-l] Re: ???

 
    Bem, ele planta a primeira muda, anda 5m, planta
a 2a, anda +5m, planta
a terceira e volta 10m até a
origem. Logo, sendo d[n] a distância percorrida p/plantar o n-ésimo
trio
de árvores, temos d[1]=20m.
    P/plantar a 4a muda, ele anda 10+5m, depois +5 p/a
5a, +5 p/a sexta
e volta +25m até a origem. Ou
seja, d[2]=50m.
    Se vc observar, d[n]=2d[n-1]+2*(5+5+5) ou seja:
d[n]=2*(d[n-1]+15).
    Assim, temos
d[1]=20
d[2]=50
d[3]=130
d[4]=290
d[5]=610
d[6]=1250
d[7]=2550  ...
Confesso que não consegui achar nenhum padrão nessa sequencia,
a não ser
que
d[n]=3d[n-1]-d[n-2]  para n>2
d[n]=8d[n-2]-3d[n-3]    para n>3
    Então, tentemos de uma forma mais simples:
os primeiros 10m ele percorre 2n vezes no trio n de mudas (10*2*n);
os 15m seguintes ele percorre 2(n-1) vezes (15*2*(n-1));
os 15m depois, 2(n-2) vezes (15*2*(n-2));
e assim sucessivamente.
    Lembrando que há 21 trios de mudas, a distância
total percorrida seria
de
D=10*2*21+15*2*20+15*2*19+...15*2*1
D=420+15*2*(20+19+18+...+1)
D=6720
    Vale lembra que, após plantar a última
muda o jardineiro NÃO precisa
voltar a origem. Portanto,
devemos excluir a distância entre a última e a primeira
muda, ou seja, precisamos
excluir 100m da conta -
se alguém não entedeu pq 100m, imagine 21 MARCOS numa
estrada, distantes
5m uns dos outros e
tente achar a distância total.
    Assim, a distância percorrida pelo nosso jardineiro
do plantio da primeira
muda até o plantio da última
muda é de 6620m ou, se vc preferir, 66,2 hectômetros(a
abreviação é hm ou
Hm?)
    Se você puder colocar a resoluçào
daquelas pessoas que acharm 65 seria
legal. Obviamente, uma das
2 está errada e a outra talvez esteja certa. De qq forma, comentem
a minha
resposta e vamos ver como
se acha 65...
[]'s
Alexandre Tessarollo
PS: NÃO tenho certeza dessa solução, ainda vou
revisá-la com a devida calma.
Achei melhor publicá-la
logo pq sei q depois de revisá-la num papel não vou ter
a necessária paciência
p/digitar... :-)) A
propósito., minhas resolução e resposta bateram
com a sua? Caso não, publique-as
vc tb!
Eder wrote:
   Olá colegas de lista, O seguinte problema,proposto
em um vestibular da
UNB,está causando uma
  controvérsia lá no colégio...Tenho um gabarito
dizendo que a resposta
é 65,porém eu não consigo chegar a
  esse resultado,nem alguns colegas de sala.Se alguém puder
resolver,agradeço.
No projeto urbanístico de
  uma cidade ,o paisagista previu a urbanização
do canteiro central de uma
das avenidas,com o plantio de
  63 mudas de Flamboyant,todas dispostas em linha reta e distantes
5m uma
da outra.No dia do plantio,o
  caminhão descarregou as mudas no canteiro central,no
local onde seria
plantada a primeira muda.Um
  jardineiro foi designado para executar o serviço.Para
isso,partindo do
lugar onde as mudas foram
  colocadas,ele pegou 3 mudas de cada vez,plantou-as nos locais
designados,enfileirando-as
uma após a
  outra.Calcule ,em hectômetros,a distância total
mínima percorrida pelo
jardineiro após finalizar o trabalho.






[obm-l] Re: ???

2002-05-01 Por tôpico Alexandre Tessarollo


Bem, ele planta a primeira muda, anda 5m, planta a 2a, anda +5m, planta
a terceira e volta 10m até a 
origem. Logo, sendo d[n] a distância percorrida p/plantar o n-ésimo trio
de árvores, temos d[1]=20m.

P/plantar a 4a muda, ele anda 10+5m, depois +5 p/a 5a, +5 p/a sexta
e volta +25m até a origem. Ou 
seja, d[2]=50m.

Se vc observar, d[n]=2d[n-1]+2*(5+5+5) ou seja:
d[n]=2*(d[n-1]+15).

Assim, temos
d[1]=20
d[2]=50
d[3]=130
d[4]=290
d[5]=610
d[6]=1250
d[7]=2550  ...

Confesso que não consegui achar nenhum padrão nessa sequencia, a não ser
que
d[n]=3d[n-1]-d[n-2]  para n>2
d[n]=8d[n-2]-3d[n-3]para n>3

Então, tentemos de uma forma mais simples:
os primeiros 10m ele percorre 2n vezes no trio n de mudas (10*2*n);
os 15m seguintes ele percorre 2(n-1) vezes (15*2*(n-1));
os 15m depois, 2(n-2) vezes (15*2*(n-2));
e assim sucessivamente.

Lembrando que há 21 trios de mudas, a distância total percorrida seria
de
D=10*2*21+15*2*20+15*2*19+...15*2*1
D=420+15*2*(20+19+18+...+1)
D=6720

Vale lembra que, após plantar a última muda o jardineiro NÃO precisa
voltar a origem. Portanto, 
devemos excluir a distância entre a última e a primeira muda, ou seja, precisamos
excluir 100m da conta - 
se alguém não entedeu pq 100m, imagine 21 MARCOS numa estrada, distantes
5m uns dos outros e 
tente achar a distância total.

Assim, a distância percorrida pelo nosso jardineiro do plantio da primeira
muda até o plantio da última 
muda é de 6620m ou, se vc preferir, 66,2 hectômetros(a abreviação é hm ou
Hm?)

Se você puder colocar a resoluçào daquelas pessoas que acharm 65 seria
legal. Obviamente, uma das 
2 está errada e a outra talvez esteja certa. De qq forma, comentem a minha
resposta e vamos ver como 
se acha 65...

[]'s

Alexandre Tessarollo

PS: NÃO tenho certeza dessa solução, ainda vou revisá-la com a devida calma.
Achei melhor publicá-la 
logo pq sei q depois de revisá-la num papel não vou ter a necessária paciência
p/digitar... :-)) A 
propósito., minhas resolução e resposta bateram com a sua? Caso não, publique-as
vc tb!

Eder wrote:

   Olá colegas de lista, O seguinte problema,proposto em um vestibular da
UNB,está causando uma 
  controvérsia lá no colégio...Tenho um gabarito dizendo que a resposta
é 65,porém eu não consigo chegar a 
  esse resultado,nem alguns colegas de sala.Se alguém puder resolver,agradeço.

No projeto urbanístico de 
  uma cidade ,o paisagista previu a urbanização do canteiro central de uma
das avenidas,com o plantio de 
  63 mudas de Flamboyant,todas dispostas em linha reta e distantes 5m uma
da outra.No dia do plantio,o 
  caminhão descarregou as mudas no canteiro central,no local onde seria
plantada a primeira muda.Um 
  jardineiro foi designado para executar o serviço.Para isso,partindo do
lugar onde as mudas foram 
  colocadas,ele pegou 3 mudas de cada vez,plantou-as nos locais 
designados,enfileirando-as
uma após a 
  outra.Calcule ,em hectômetros,a distância total mínima percorrida pelo
jardineiro após finalizar o trabalho.



=
Instruções para entrar na lista, sair da lista e usar a lista em
http://www.mat.puc-rio.br/~nicolau/olimp/obm-l.html
O administrador desta lista é <[EMAIL PROTECTED]>
=



Re: [obm-l] Re:JP e a todos. (extensão)

2002-04-25 Por tôpico Alexandre Tessarollo


Extendendo o desafio do seu professor, ligue os pontos D, M e N da figura
do Rafael. Todos os triângulos 1, 2 e 3 de antes foram dividos em duas partes,
sendo uma "mais interna" visivelmente menor e outra "mais externa"
visivelmente maior. Quero o seguinte:
1) Prove que as áreas dos triângulos maiores ("mais externos") são iguais
entre si;
2) idem para os menores ("mais internos");
3) Se a área de um "menor" for X, qual a área de um "maior"?

[]'s

Alexandre Tessarollo

Rafael WC wrote:

> > 1-Prove que em um triângulo qualquer , suas medianas
> > o divide em outros
> > 6 triângulos  de mesma área.
>
> Olá Rick!
>
> Espero que a figura siga junto...
>
> Nessa figura, coloquei os números em alguns triângulos
> para identificarmos algumas áreas. Repare que coloquei
> o número 1 nos triângulos BDE, BDG e CDG. Você pode
> ver que a área dos triângulo BDG e CDG são iguais,
> pois os dois triângulos têm a mesma base (BD = DC) e
> os dois têm a mesma altura (distância do ponto G até o
> lado BC).
>
> Feito isso, também podemos concluir a mesma coisa com
> relação aos triângulos AMG e CMG, pelo mesmo motivo,
> as bases são iguais e a altura é a mesma. Por isso,
> assinalei esses dois triângulos com o nº 2.
>
> E da mesma forma, temos que BGN e AGN têm a mesma
> área. E foram assinalados com o nº 3.
>
> Agora repare também que, o triângulo ADB tem a mesma
> área do triângulo ADC, pois ambos têm a mesma base e a
> mesma altura. Ou seja, cada um tem metade da área do
> triângulo ABC. Então vamos escrever essas áreas, em
> função dos números. Vou chamar as áreas que estão com
> o nº 1 de A1, as com o nº2 de A2 e as outras de A3:
> ADB = ADC
> BDG + BGN + GNA = DCG + CGM + GMA
> A1 + A3 + A3 = A1 + A2 + A2
> A3 + A3 = A2 + A2
> 2.A3 = 2.A2
> A3 = A2
>
> Agora podemos também ver que as áreas dos triângulos
> BMA e BMC são iguais, pois os dois têm a mesma base
> (MC = MA) e a mesma altura (distância do vértice B ao
> lado AC). Então escrevendo em função das áreas dos
> triângulos menores:
> BMA = BMC
> MAG + ANG + NGB = GCM + GCD + BGD
> A2 + A3 + A3 = A2 + A1 + A1
> A3 + A3 = A1 + A1
> 2.A3 = 2.A1
> A3 = A1
>
> Como A1 = A3 e A3 = A2, concluímos que:
> A1 = A2 = A3
>
> Então, as 6 áreas em que ficou dividido o triângulo
> ABC são todas iguais e equivalem a 1/6 da área total
> do triângulo ABC cada uma.
>
> > Um abraço..
> > Rick Barbosa
> >
> Outro,
>
> Rafael.
>
> =
> Rafael Werneck Cinoto
>ICQ# 107011599
>  [EMAIL PROTECTED]
>[EMAIL PROTECTED]
>[EMAIL PROTECTED]
> http://www.rwcinoto.hpg.com.br/
>
> __
> Do You Yahoo!?
> Yahoo! Games - play chess, backgammon, pool and more
> http://games.yahoo.com/
>
>   
>  [Image]

=
Instruções para entrar na lista, sair da lista e usar a lista em
http://www.mat.puc-rio.br/~nicolau/olimp/obm-l.html
O administrador desta lista é <[EMAIL PROTECTED]>
=



[obm-l] Geometria dos balcãs

2002-04-23 Por tôpico Alexandre Tessarollo


Da outra lista, um pouco de diversão...

[]'s

Alexandre Tessarollo

PS: Ainda não so li com a devida calma, mas acho que falta uma parte do
enunciado do primeiro prob retirado da 3a olimpíada...


==

   Date: Tue, 23 Apr 2002 02:41:41 -0700
   From: "Antreas P. Hatzipolakis" <[EMAIL PROTECTED]>
Subject: BMO

The Balkan Mathematical Olympiads:

 1st - 16th: http://bmo.ournet.md/previous.html

   17th: http://bmo.ournet.md/index.html

Geometry Problems (selection):

Let O be the circumcenter of a triangle ABC, D be the midpoint of
the side AB and E be the centroid of the triangle ACD. Prove that
the lines CD and OE are orthogonal if and only if AB=AC.

   The 2nd Balkan Mathematical Olympiad
   1985, Sofia, Bulgaria

---

A line that passes through the incenter I of the triangle ABC meets
the incircle in D and E and the circumcircle of the triangle ABC in
F and G (D is between I and F). Prove that , where r is the radius
of the incircle. When does the equality hold?

Let ABCD be a tetrahedron and points E, F, G, H, K, L be situated
on the edges AB, BC, CA, DA, DB, DC respectively. Prove that if

   AE*BE = BF*CF = CG*AG = DH*AH = DK*BK = DL*CL

then the points E, F, G, H, K, L are placed on a sphere.

   The 3rd Balkan Mathematical Olympiad
   1986, Bucharest, Romania

---

Let A1B1C1 be the orthic triangle of an acute-angled nonequilateral
triangle ABC and A2, B2, C2 be the contacts of the incircle of the
triangle A1B1C1 with its sides. Prove that the triangles A2B2C2 and
ABC have the same Euler line.

  The 7th Balkan Mathematical Olympiad
  1990, Sofia, Bulgaria

---

Three circles Gamma, C1 and C2 are given in the plane. C1 and
C2 tangent Gamma internally at points B and C, respectively.
Moreover C1 and C2 tangent each other externally at a point D.
Let A be one point in which the common tangent of C1 and C2
intersects Gamma. Denote by M the second point of intersection
of the line AB and the circle C1 and by N the second point of
intersection of the line AC and the circle C2. Further denote by
K and L second points of intersections of the line BC with C1
and C2, respectively. Show that lines AD, MK and NL are
concurrent.

   The 14th Balkan Mathematical Olympiad
   Kalampaka, Greece, April 29, 1997

---

Given an acute triangle ABC, let D be the midpoint of the arc BC of
the circumcircle around the triangle ABC, not containing the point A.
The points which are symmetric to D with respect to the line BC and
the circumcentre O are denoted by E and F, respectively. Finally,
let K be the midpoint of the segment EA. Prove that:
  a) The circle, passing through the midpoints of the sides of
the triangle ABC, also passes through K;
  b) The line, passing through K and the midpoint of the segment
BC is perpendicular to the line AF.

  The 16th Balkan Mathematical Olympiad
  Ohrid, FYR Macedonia, May 7th, 1999

---

APH

=
Instruções para entrar na lista, sair da lista e usar a lista em
http://www.mat.puc-rio.br/~nicolau/olimp/obm-l.html
O administrador desta lista é <[EMAIL PROTECTED]>
=



Re: [obm-l] Estudos sobre Equações

2002-04-23 Por tôpico Alexandre Tessarollo



[EMAIL PROTECTED] wrote:

> Olá amigos..
> Ai vão alguns problemas interessantes de equações..
> Se puderem me dar uma luz...
>
> 1-
> O número de raízes reais da equação
> x.(x + 1).(x² + x + 1) = 42

>
>
> 2-
> O número de raízes reais da equação
> 3x^4 - 2x³ + 4x² - 4x + 12 = 0

>
>
> Há para essas equações 1 e 2 alguma critério ?
>
> 3-
> A diferença entre a maior e a menor raiz da equação
> (x - 2)(x - 3)(x - 4)(x - 5 ) = 360
>

Admitindo que exista x inteiro como solução, temos que escrever 360 como
produto de 4 inteiros consecutivos.
1*2*3*4=24
2*3*4*5=120
3*4*5*6=360Logo, uma raiz é x=8.

Vale lembrar que (-3)*(-4)*(-5)*(-6)=360, logo, x=-1 tb é raiz. Se vc
desenvolver o produto e arrumar a eq, vc deverá ter
x^4-14x^3+71x^2-154x-240=0.
Agora que já conhecemos duas raízes, podemos baixar o grau dessa eq até um de
segundo grau, que deverá ser x^2-7x+30=0. Observando essa eq, ela só admite
raízes complexas.

Logo, às raízes de maior e menor valor são, respectivamente, 8 e -1, e
sua diferença é 9. Todavia, deve haver uma forma melhor de garantir que as
outras duas raízes são complexas... Alguém se habilita?




>
> 4-
> A diferença entre a maior e a menor raiz da equação
> (x² + x + 1)(2x² + 2x + 3 ) = 3(1 - x - x²)

Hum... De olho só sei dizer que zero é raiz... No braço, sei dizer que só
existe uma outra raiz real e que ela está entre -2 e -1, se eu não errei as
contas. Mas não sei como tirar essa outra ou como mostrar, de forma
"tragável" que as outras dua raízes são complexas...

>
>
> São todos exercícios muito bons , com conhecimento a nível de 1° grau ,
> eu não consegui enxergar uma solução válida.

Ih! Oops... Só valia 1º grau? Hum... Bem, isso não permite muita coisa...
Contudo, a solução da 3 ainda é "válida", posto q eles já sabem Báskara (ou
método de resolução de eqs do 2º grau, p/os puristas ;-)) O problema é q um
aluno de 1o grau só vai ter duas raízes e não vai sequer ter noção de q
existem outras Na grosseria e na inocência, ele vai poder responder a
pergunta da questão...


>
> Obrigado..
> Rick Barbosa

[]'s

Alexandre Tessarollo

=
Instruções para entrar na lista, sair da lista e usar a lista em
http://www.mat.puc-rio.br/~nicolau/olimp/obm-l.html
O administrador desta lista é <[EMAIL PROTECTED]>
=



Re: [obm-l] Resolução Geo Plana exerc. 3

2002-04-19 Por tôpico Alexandre Tessarollo



[EMAIL PROTECTED] wrote:

> Aderbal,aí vai novamente a resolução, se algum coléga da lista puder opinar
> ...
> Grato..
>
> Como AP = PD = PE
>
> Lado do   Q(ABCD) = 10 + h(APD)

L=10+h
h=L-10

Portanto, teremos um triângulo retângulo de hipotenusa PD=10 e catetos h=L-10 e
L/2. Fazendo o Pitágorás, fica:

10^2=(L-10)^2+(L/2)^2
100=L^2+100-20L+L^2/4
0=5L^2/4-20L
5L(L/4-4)=0

L/4-4=0
L/4=4
L=16


Alguém concorda? Alguém discorda? Alguém não entendeu o que eu fiz?
Manifestem-se...

[]'s

Alexandre Tessarollo

=
Instruções para entrar na lista, sair da lista e usar a lista em
http://www.mat.puc-rio.br/~nicolau/olimp/obm-l.html
O administrador desta lista é <[EMAIL PROTECTED]>
=



Re: [obm-l] Equação do 2°

2002-04-19 Por tôpico Alexandre Tessarollo

Bem, sabemos que as raízes são (-b+sqrt(delta))/2ae
(-b-sqrt(delta))/2a. Trabalhando só nos reais, podemos dizer que a primeira é
sempre maior ou igual à segunda. Logo, a diferença das duas é:

(-b+sqrt(delta))/2a-(-b-sqrt(delta))/2a=
=(-b+sqrt(delta)+b+sqrt(delta))/2a=
=2sqrt(delta)/2a=
sqrt(delta)/a


Na equação específica que vc pediu, fica

sqrt[(2+sqrt3)^2-4(7+4sqrt3)(-2)]/(7+4sqrt3) =

=sqrt[4+3+4sqrt3+56+32sqrt3]/(7+4sqrt3)=
=sqrt[63+36sqrt3]/(7+4sqrt3)=
=3sqrt[7+4sqrt3]/(7+4sqrt3)

Esse resultado até é bonitinho, mas se vc quiser racionalizar, fica

3sqrt[7+4sqrt3](7+4sqrt3)

Bem, se eu não errei nenhuma conta, é isso aí... Confiram!

[]'s

Alexandre Tessarollo



(7 + 4 sqrt3)x^2 +(2 + sqrt3)x - 2 = 0

[EMAIL PROTECTED] wrote:

> Olá amigos..
>
> Caro Aderbal..
> A seguinte questão do quadrado que possui um ponto interior que dista 10
> cm de dois vértices e 10 cm do lado , a resolução que eu lhe mandei ,acho
> que esta correta ?Mais posso ter me enganado em algum lugar ..mais a idéia
> , acho que é mais ou menos como esta lá..
> Vou tentar lhe mandar a figura com tudo direitinho .
>
> Rafael, obrigado pela dica , vou tentar passar a figura para esse formato.
>
> E para não perder a viagem , vai ai um exercício de equação do 2°.
>
> 1-A diferença entre a maior e a menor raiz da equação (7 + 4 sqrt3)x^2 +
> (2 + sqrt3)x - 2 = 0
>
> Obs:
> Eu tentei fazer pela Soma e pelo Produto , tipo:
> Achava a soma o produto , e depois montava um sistema com eles , explicitava
> na soma x1 e substituía no produto , para encontrar x2 , tentei fazer no
> braço , mais não consegui .
> Será que existe alguma relação entre a diferença de raízes ?(Estilo a Soma
> -b/a e produto c/a)
> Grato..
> Rick Barbosa
>
> --
> Use o melhor sistema de busca da Internet
> Radar UOL - http://www.radaruol.com.br
>
> =
> Instruções para entrar na lista, sair da lista e usar a lista em
> http://www.mat.puc-rio.br/~nicolau/olimp/obm-l.html
> O administrador desta lista é <[EMAIL PROTECTED]>
> =

=
Instruções para entrar na lista, sair da lista e usar a lista em
http://www.mat.puc-rio.br/~nicolau/olimp/obm-l.html
O administrador desta lista é <[EMAIL PROTECTED]>
=



[obm-l] Re:

2002-04-18 Por tôpico Alexandre Tessarollo



Adherbal Rocha Filho wrote:

>   Olá pessoal,
> Vocês poderiam me ajudar com ests questões?
>
> 1. P é um pnt interior a um quadrado ABCD.As distancias de P aos
> verices A e D e ao lado BC são iguais a 10.O lado do quadrado mede?

Sugiro uma figura, mas se vc estiver com preguiça, o ASCII deve quebrar
um galho

AB

E   PF

D C

E é o ponto médio de AD. Como AP=DP, podemos afirmar que P está na
mediatriz de AD e, logo, EP é perpendicular a AD. F é a intersecção de EP com
BC. Sabmeos que AP=DP=PF=10.

Pela figura, é fácil ver que o lado do quadrado x=AB=EP+PF=EP+10. Logo,
x=EP+10; EP=10-x. AE=x/2. Por Pitágoras, temos:

AE^2+EP^2=AP^2
(x/2)^2+(10-x)^2=10^2
x^2/4+100+x^2-20x=100
5x^2/4-20x=0
5x(x/4-4)=0

x/4-4=0
x=16



>
>
> 2. Só mais esta:
> Determine as soluções reais de x^2=2^x
> Obrigado!
>

Essa eu já vi diversas similares mas até hoje não aprendi a fazer esse
tipo de questão... Mas, se for te ajudar, x=2 é uma soluçào "óbvia" do
equação. Olhando pelo gráfico de x^2 e 2^x (um tanto similar a da exp(x)),
vemos que eles se cortam em apenas dois pontos. Resta agora achar o outro.

Parêntesis
Momento infame e infeliz daquele professor q não sabe responder: "Pronto,
já resolvi metade do prob com x=2 e indiquei o caminho para a segunda raiz.
Agora o resto é com vc"
Fim do(s) parêntesis

[]'s

Alexandre Tessarollo

=
Instruções para entrar na lista, sair da lista e usar a lista em
http://www.mat.puc-rio.br/~nicolau/olimp/obm-l.html
O administrador desta lista é <[EMAIL PROTECTED]>
=



[obm-l] Número neperiano

2002-04-18 Por tôpico Alexandre Tessarollo


Como se explica o que é número neperiano p/um aluno do 3º ano do
Ensino Médio (ou seja, um vrestibulando)? Vale lembrar que o sujeito NÃO
está familiarizado com log e NUNCA viu exp... Obviamente, dizer q é a
base de log resultante da integral de 1/x tb não vale :0)

[]'s

Alexandre Tessarollo

=
Instruções para entrar na lista, sair da lista e usar a lista em
http://www.mat.puc-rio.br/~nicolau/olimp/obm-l.html
O administrador desta lista é <[EMAIL PROTECTED]>
=



Re: [obm-l] TN

2002-04-11 Por tôpico Alexandre Tessarollo


Responde sim, obrigado. E desculpas a todos pelo "reçiproca"... :-)

[]'s

Alexandre Tessarollo

Luiz Alberto Duran Salomao wrote:

> Caro Alexandre:
> A reciproca nao eh verdadeira.
> Contra-exemplo: 111 eh composto (pois eh divisivel por 3) mas 3 nao eh
> composto.
> Acho que isto responde tambem aas suas outras questoes, nao eh ?
> Abracos,
> Luiz Alberto Salomao
>
> Alexandre Tessarollo wrote:
>
> > Luiz Alberto Duran Salomao wrote:
> >
> > > Caros amigos: Este exercicio foi enviado para a lista jah faz algum
> > > tempo, mas parece-me
> > > que ainda nao foi respondida.
> > >
> >
> > [...]
> >
> > >
> > > Um abraco a todos,
> > > Luiz Alberto Salomao
> > >
> > > Rubens Vilhena wrote:
> > >
> > >> Olá, pessoal! Espero que me ajudem em minhas dúvidas sobre  Números
> > >> Inteiros. 1) Se n é composto então o número 11111 (n vezes)
> > >> também é composto. Obrigado!
> > >>
> > >
> > 2,5 dúvidas. Primeira: a reçiproca é verdadeira? Ou seja, se
> > 111...11 (n vezes) é composto, então n tb é composto? Segunda: E com n
> > primo? 111...11 (n vezes) é primo ou é composto? 2,5-ésima dúvida: E a
> > reciproca (da segunda dúvida), é verdadeira?
> >
> > []'s
> >
> > Alexandre Tessarollo
> >
> > =
> > Instruções para entrar na lista, sair da lista e usar a lista em
> > http://www.mat.puc-rio.br/~nicolau/olimp/obm-l.html
> > O administrador desta lista é <[EMAIL PROTECTED]>
> > =
>
> =
> Instruções para entrar na lista, sair da lista e usar a lista em
> http://www.mat.puc-rio.br/~nicolau/olimp/obm-l.html
> O administrador desta lista é <[EMAIL PROTECTED]>
> =

=
Instruções para entrar na lista, sair da lista e usar a lista em
http://www.mat.puc-rio.br/~nicolau/olimp/obm-l.html
O administrador desta lista é <[EMAIL PROTECTED]>
=



Re: [obm-l] TN

2002-04-10 Por tôpico Alexandre Tessarollo



Luiz Alberto Duran Salomao wrote:

> Caros amigos: Este exercicio foi enviado para a lista jah faz algum
> tempo, mas parece-me
> que ainda nao foi respondida.
>

[...]


>
> Um abraco a todos,
> Luiz Alberto Salomao
>
> Rubens Vilhena wrote:
>
>> Olá, pessoal! Espero que me ajudem em minhas dúvidas sobre  Números
>> Inteiros. 1) Se n é composto então o número 11111 (n vezes)
>> também é composto. Obrigado!
>>
>
2,5 dúvidas. Primeira: a reçiproca é verdadeira? Ou seja, se
111...11 (n vezes) é composto, então n tb é composto? Segunda: E com n
primo? 111...11 (n vezes) é primo ou é composto? 2,5-ésima dúvida: E a
reciproca (da segunda dúvida), é verdadeira?

[]'s

Alexandre Tessarollo

=
Instruções para entrar na lista, sair da lista e usar a lista em
http://www.mat.puc-rio.br/~nicolau/olimp/obm-l.html
O administrador desta lista é <[EMAIL PROTECTED]>
=



Re: [obm-l] duvida

2002-04-10 Por tôpico Alexandre Tessarollo


Para simplifcar a vida, vou pensar numa cidade de 100 hab e vou
chamar
H (homens)/M (mulheres);
J ("jovens" com menos de 30)/I ("idosos" com +de 30);
S (solteiro)/C (casado).


 (unirio-rj) Tendo  sido feito o censo populacional 96 em uma cidade ,
descobriu-se sobre a população que:
1) 44% têm idade superior a 30 anos;
HIC+HIS+MIC+MIS=44
 2) 68% são homens ;
HJS+HJC+HIS+HIC=68
 3) 37% são homens com mais de 30 anos
HIC+HIS=37
 4) 25% são homens solteiros
HIS+HJS=25
 5) 4% são homens solteiros com mais de 30 anos
HIS=4
 6) 6%  são individuos  solteiros com mais de 30 anos
HIS+MIS=6
 com base nos dados anteriores ,pode-se afirmar q a porcentagem  da
população  desta  cidade q representa as mulheres casadas com idade
igual ou inferior a 30 anos é de?
MJC=?

Aplicando 5 em 3, 4 e 6, teremos, respectivamente: HIC=33; HJS=25 e
MIS=2. Aplicando esses resultados em 2, teremos HJC=6. Com isso já
calculamos todo o grupo dos homens e tb sabemos MIS. Com esses valores e
a equação 1, temos MIC=5. Hum... se somarmos td teremos q ter 100 (ou,
por 2, sabemos q só as mulheres somam 32), veremos que MJS+MJC=25. Bem,
aí não sei mais sair Creio q faltou algum informação... Alguém se
habilita? Abaixo os valores q encontrei, organizados.
 HJS = 25
HJC = 6
HIS = 4
HIC = 33
MJS = ?
MJC = 25-?
MIS = 2
MIC = 5

[]'s

Alexandre Tessarollo

=
Instruções para entrar na lista, sair da lista e usar a lista em
http://www.mat.puc-rio.br/~nicolau/olimp/obm-l.html
O administrador desta lista é <[EMAIL PROTECTED]>
=



[obm-l] Re: (a+bi)^(c+di)

2002-04-02 Por tôpico Alexandre Tessarollo



Agradeço às respostas sucintas do N e do Morgado e em particular, à "prolixa"
do JP :-)



Mas restou uma dúvida: se z=r*cis(t), então ln(z)=ln(r)+i*(t+2kpi). Foi
dito que 
z^w=e^(w*ln(z))=e^(c+d*i)*(ln(r)+i*(t+2kPI))=e^{[c*ln(r)-d*t-d*2kPI]+i[c*t-c*2kPI+d*ln(r)]}

Chamando X=c*ln(r)-d*t-d*2kPI e Y=c*t-c*2kPI+d*ln(r), temos 
z^w=e^(X+iY)=(e^X)*(e^iY)=(e^X)*((e^i)^Y)

Obviamente X e Y dependem do valor de k (do 2kPI). Mesmo assim, eu sei
calcular e^x. Só que eu não sei quanto vale e^i. Mesmo que seja um número
complexo, sei elevá-lo à Y, mas preciso saber como calcular e^i...

Imagino que deva sair pela série ou por outro caminho, mas nos meus rascunhos,
e^i pela série resulta em 
somatório [zero a infinito] {[16k^2+12k+1]*[4k+3+i]/(4k+3)!}

Como vcs podem ver, ficou meio feio... Como dizia o poeta, "E agora, José?"
(com todos os trocadilhos, JP :-))))


[]'s

Alexandre Tessarollo



=
Instruções para entrar na lista, sair da lista e usar a lista em
http://www.mat.puc-rio.br/~nicolau/olimp/obm-l.html
O administrador desta lista é <[EMAIL PROTECTED]>
=



Re: [obm-l] area do triângulo + erro

2002-03-31 Por tôpico Alexandre Tessarollo



Siberia Olympia wrote:

> Caros amigos, gostaria de ajuda na questâo
>
>Seja ABCDE um pentágono de lados AB, BC, CD, DE e EA tal que
> Area(ABC)=Area(ABD)=Area(ACD)=Area(ADE)=17. Calcular a medida da área do
> triângulo BCE.
>
>   Um forte abraço, cg.

Só por diversão, vamos construir a figura :-) Coloque A, B e C quaisquer
no plano. . Se ABC=ABD, então D está numa paralela à AB que passa por C ou no
reflexo desta paralela em relação à AB. Se ABD=ACD, então

D1 = C + vetor(AB)
D2 = C - vetor(AB)
D3 = intersecção da reta refletida com uma paralela à BC passando por A

Se ABD=ADE, então E está na paralela à AD passando por B ou no reflexo
desta em relação à AD. Como A, D2 e D3 estão alinhados, D2 e D3 dão o mesmo
par de retas para E: a reta Bc e o reflexo desta com relação à AD2 (AD3). Já
D1 dá uma passando por C e outra por B, mas como ADB=ADC, então essas
paralelas são equidistantes de AD1, logo não há problemas.

Portanto, temos dados A, B e C, temos 3 opções para D, depois disso,
colocar o E em qualquer lugar de duas paralelas à AD.


Se escolhermos D2 ou D3, não poderemos colocar E sobre a reta BC ou
área(BCE)=0. Assim, coloquemos sobre o reflexo de BC em relação à AD2 (AD3).
área(BCE) = BC*(dist[E,BC])
Como A equidista da reta em que está E e da reta BC, temos que
dist[E,BC]=2*dist[A,BC]. Logo:
área(BCE) = 2*BC*(dist[A,BC]) = 2*área(ABC) = 34


No entanto, se escolhermos D1, a área de BCE poderá variar de zero a
infinito. Alguém sabe explicar qual o erro em D1?? Tudo leva a crer que D2 e
D3 estejam certos, mas qual o erro de D1? Na figura é fácil ver que o único
ponto E que dá a mesma resposta é (a intersecção de CD1 com a paralela à AD1
que passa por B) ou, equivalentemente, (D1+vetor(AB)). Ah sim, tb há o
reflexo deste ponto em relação à BC. E aí, alguém se habilita??


[]'s

Alexandre Tessarollo

=
Instruções para entrar na lista, sair da lista e usar a lista em
http://www.mat.puc-rio.br/~nicolau/olimp/obm-l.html
O administrador desta lista é <[EMAIL PROTECTED]>
=



[obm-l] (a+bi)^(c+di)

2002-03-30 Por tôpico Alexandre Tessarollo

Esta é para todos, em especial o JP e o séquito de "seguidores" que se forma
aqui na lista :-)

  Como faço para elevar um número complexo qualquer a outro? Sei que isso
é possível, mas ainda não vi essa parte na fac... Seria possível pelo menos
uma dica ou idéia de como se faz? Alguma bibliografia?

[]´s e Feliz Páscoa!!

Alexandre Tessarollo



=
Instruções para entrar na lista, sair da lista e usar a lista em
http://www.mat.puc-rio.br/~nicolau/olimp/obm-l.html
O administrador desta lista é <[EMAIL PROTECTED]>
=



Re: [obm-l] Novov grupo

2002-03-26 Por tôpico Alexandre Tessarollo



[EMAIL PROTECTED] wrote:

> Fiquei sabendo de um novo grupo de discussões que é muito bom e trata de
> todos os assuntos relacionados às exatas, indo desde a Matemática superior
> até a Engenharia em geral.

Bem, ia perguntar qual o endereço desse grupo, mas vc já deu (em outra
msg) :-))

Notei que vc mandou essa msg não só p/a OBM-1 como para um
"[EMAIL PROTECTED]". Gostaria de saber mais sobre esse grupo
tb. Depois manda p/a lista (OBM) as instruções p/inscrição e/ou uma home-page
com as descrições de cada lista, por favor.


>
> Vale a pena!
> Um abraço!

[]'s

Alexandre Tessarollo


=
Instruções para entrar na lista, sair da lista e usar a lista em
http://www.mat.puc-rio.br/~nicolau/olimp/obm-l.html
O administrador desta lista é <[EMAIL PROTECTED]>
=



Re: [obm-l] Álgebra

2002-03-22 Por tôpico Alexandre Tessarollo


Hum... 133^5 termina em 3; 110^5 termina em 0; 84^5 termina em 4; 27^5
termina em 7; logo, n^5 termina em 3+0+4+7 q termina em 4. Portanto, o único
algarismo p/a unidade de n é 4 - basta ver q SÓ os números terminados em 4,
qundo elevados a 5, terminam em 4. Tente ver se sai agora... Se não me engano
tinha um "truque" de Ágebra I que toernava este prob MUITO fácil, mas não
estou lembrando agora. Se eu achar antes de alguém aqui responder eu mando.

[]'s

Alexandre Tessarollo

Rafael WC wrote:

> Olá Pessoal!
>
> Gostaria de saber uma resolução para esse exercício:
> O inteiro positivo "n", tal que
> 133^5 + 110^5 + 84^5 + 27^5 = n^5
>
> é igual a...
>
> A resposta é n = 144, mas e a resolução???
>
> Obrigado,
>
> Rafael.
>
> =
> Rafael Werneck Cinoto
>   ICQ# 107011599
> [EMAIL PROTECTED]
>   [EMAIL PROTECTED]
>   [EMAIL PROTECTED]
> http://www.rwcinoto.hpg.com.br/
>
> __
> Do You Yahoo!?
> Yahoo! Movies - coverage of the 74th Academy Awards®
> http://movies.yahoo.com/
> =
> Instruções para entrar na lista, sair da lista e usar a lista em
> http://www.mat.puc-rio.br/~nicolau/olimp/obm-l.html
> O administrador desta lista é <[EMAIL PROTECTED]>
> =


=
Instruções para entrar na lista, sair da lista e usar a lista em
http://www.mat.puc-rio.br/~nicolau/olimp/obm-l.html
O administrador desta lista é <[EMAIL PROTECTED]>
=



Re: [obm-l] Problema de Geometria

2002-03-10 Por tôpico Alexandre Tessarollo



Mensagem original

>Calcular a área de um triângulo retângulo de perímetro 2p e altura relativa
>a hipotenusa h.
>
>=
>Instruções para entrar na lista, sair da lista e usar a lista em
>http://www.mat.puc-rio.br/~nicolau/olimp/obm-l.html
>O administrador desta lista é <[EMAIL PROTECTED]>
>=
>


   Hum... Bem, por preguiça vou chamar o perímetro de P mesmo... "a" é a
hipotenusa.

a+b+c=P
b+c=P+a
b^2+c^2+2bc=P^2+a^2-2aP(I)

a^2=b^2+c^2(II)

ah=bc  (III)

Aplicando II em I, temos:
a^2+2bc=P^2+a^2-2aP
2bc=P^2-2aP

Aplicando III, temos
2ah=P^2-2aP
a=P^2/(2(h-P)) (IV)

Note que II pode ser reescrita como
a^2=(b+c)^2-2bc

Aplicando III, temos
a^2=(b+c)^2-2ah
2ah=(b+c)^2-a^2
2ah=(b+c+a)(b+c-a)

Lembrando que P=a+b+c, temos
2ah=P(P-2a)

Lembre que a área S procurada pode ser escrita como S=ah/2. Assim, temos:
4S=P(P-2a)

Aplicando IV, temos:
4S=P(P-2(P^2/(2(h-P
4S=P(P-(P^2/(h-P)))
4S=P(P+P^2/(P-h))

Rearrumando para ficar mais simpático:

4S=P(2P^2-hP)/(P-h)
S=P(2P^2-hP)/(4(P-h))

Se eu não errei nenhuma conta, é isso aí... Espero ter ajudado...

[]'s

Alexandre Tessarollo




=
Instruções para entrar na lista, sair da lista e usar a lista em
http://www.mat.puc-rio.br/~nicolau/olimp/obm-l.html
O administrador desta lista é <[EMAIL PROTECTED]>
=



Re: [obm-l] Angulo de 1 grau

2002-03-09 Por tôpico Alexandre Tessarollo



Jose Paulo Carneiro wrote:

> Se 1 grau fosse construtivel, entao 20 graus tambem seria (facil).
> No entanto, 20 graus nao eh construtivel, como estah em todos os livros
> (por exemplo, Construcoes geometricas, de E.Wagner, apendice).
> JP

Não sei se me expressei bem, mas o problema de 19 me fez pensar no de 1.
Quis colocar um problema SEPARADO do de 19, OUTRO problema, OUTRA discussão.
O de 19 serviu apenas p/me "inspirar".. :-))

Respondendo à resposta do Nicolau:

Vou olhar com calma a sua resposta, mas gostaria de obter algo mais
geométrico, se possível, puramente geométrico, apenas régua e compasso. Se
for o caso, "vale" incluir lugares geométricos construtíveis ponto-a-ponto,
como uma hipérbole, por exemplo. Afinal, Cabri, SketchPad e similares estão
aí p/isso  :-)

(OBS: Quem já conhece, saiba que o SkewtchPad 4 já possui uma versão demo
disponível p/downloads em www.keypress.com/sketchpad)

[]'s

Alexandre Tessarollo

>
>
> - Original Message -
> From: Alexandre Tessarollo <[EMAIL PROTECTED]>
> To: <[EMAIL PROTECTED]>
> Sent: Saturday, March 09, 2002 2:27 AM
> Subject: [obm-l] Angulo de 1 grau
>
> Estava olhando um prob que propuseram na lista (DADO um ângulo de
> 19, construir o de 1) e lembrei que o ângulo de 1 não é construtível,
> mas não lembro como provar... Alguém se habilita?
>
> Além disso, quais são os ângulos  construtíveis por régua e
> compasso? Refiro-me aos ângulos de valores inteiros em graus, como por
> exemplo 15, 30, 45, 60
>
> Para finalizar, sei que podemos bissectar um ângulo mas não
> trisectá-lo. É possível dividí-lo em n partes, n NÃO sendo uma potência
> de 2?
>
> Aliás, na faculdade um professora minha até me mostrou uma maneira
> de trissectar usando régua ESCALONADA e compasso. Mas foi só um
> comentário rápido e já se vai um bom tempo... Alguém?
>
> []'s
>
> Alexandre Tessarollo
>
> =
> Instruções para entrar na lista, sair da lista e usar a lista em
> http://www.mat.puc-rio.br/~nicolau/olimp/obm-l.html
> O administrador desta lista é <[EMAIL PROTECTED]>
> =
>
> =
> Instruções para entrar na lista, sair da lista e usar a lista em
> http://www.mat.puc-rio.br/~nicolau/olimp/obm-l.html
> O administrador desta lista é <[EMAIL PROTECTED]>
> =


=
Instruções para entrar na lista, sair da lista e usar a lista em
http://www.mat.puc-rio.br/~nicolau/olimp/obm-l.html
O administrador desta lista é <[EMAIL PROTECTED]>
=



[obm-l] Angulo de 1 grau

2002-03-08 Por tôpico Alexandre Tessarollo



Estava olhando um prob que propuseram na lista (DADO um ângulo de
19, construir o de 1) e lembrei que o ângulo de 1 não é construtível,
mas não lembro como provar... Alguém se habilita?

Além disso, quais são os ângulos  construtíveis por régua e
compasso? Refiro-me aos ângulos de valores inteiros em graus, como por
exemplo 15, 30, 45, 60

Para finalizar, sei que podemos bissectar um ângulo mas não
trisectá-lo. É possível dividí-lo em n partes, n NÃO sendo uma potência
de 2?

Aliás, na faculdade um professora minha até me mostrou uma maneira
de trissectar usando régua ESCALONADA e compasso. Mas foi só um
comentário rápido e já se vai um bom tempo... Alguém?

[]'s

Alexandre Tessarollo


=
Instruções para entrar na lista, sair da lista e usar a lista em
http://www.mat.puc-rio.br/~nicolau/olimp/obm-l.html
O administrador desta lista é <[EMAIL PROTECTED]>
=



Re: [obm-l] Provar q o produto de 3 nr consecutivos...

2002-03-08 Por tôpico Alexandre Tessarollo


Hum... Bem, se são 3 números consecutivos, então ou o segundo é par ou os
outros dois são. Produto de um par opor qualquer número inteiro é sempre par.
(No fundo, é a mesma explicação do Hugo, mas sem conta :-))

[]'s

Alexandre Tessarollo

Hugo Iver Vasconcelos Goncalves wrote:

> (x-1)*x*(x+1)
> x^3 - x
>
> sejam os numeros pares da forma 2k e os ímpares da forma 2k+1.
>
> se x par então:
> (2k)^3 - 2k
> 2(4k^3 - k) .
>
> se x ímpar então:
> (2k + 1)^3 - (2k+1)
> 2(4k^3 + 6k^2 +2k) .
>
> []´s hugo
> - Original Message -
> From: "DC" <[EMAIL PROTECTED]>
> To: <[EMAIL PROTECTED]>
> Sent: Friday, March 08, 2002 2:03 PM
> Subject: [obm-l] Provar q o produto de 3 nr consecutivos...
>
> Oi pessoal,
>
>   Preciso de uma ajuda:
> Prove que o produto de 3 numeros inteiros consecutivos
> é par.
>
>   Amplexos,
>
>   Douglas Fabiano Drumond de Carvalho
> 
> UAI - ESTADO DE MINAS
> O Grande portal dos Mineiros na Internet
> http://www.uai.com.br
> =
> Instruções para entrar na lista, sair da lista e usar a lista em
> http://www.mat.puc-rio.br/~nicolau/olimp/obm-l.html
> O administrador desta lista é <[EMAIL PROTECTED]>
> =
>
> =
> Instruções para entrar na lista, sair da lista e usar a lista em
> http://www.mat.puc-rio.br/~nicolau/olimp/obm-l.html
> O administrador desta lista é <[EMAIL PROTECTED]>
> =


=
Instruções para entrar na lista, sair da lista e usar a lista em
http://www.mat.puc-rio.br/~nicolau/olimp/obm-l.html
O administrador desta lista é <[EMAIL PROTECTED]>
=



Re: [obm-l] Sebos

2002-02-20 Por tôpico Alexandre Tessarollo

Eduardo Quintas wrote:

> Olá pessoal
> Gostaria de saber endereços de bons sebos na cidade do rio de janeiro...

A Livraria São José, no Centro, é uma ótima. Uns tempos atrás recebeu
a
coleção de um porfessor da Uerj(?) que havia falecido. Não estou me lembrando
do
endereço agora, mas depois ponho aqui na lista. Se alguém lembrar antes...


>
> Estou procurando o livro do caronet de áreas - tomo iv e não sei onde
posso
> conseguir será que tem na biblioteca do impa ?

A biblioteca do Impa está online. Basta ir ao site www.impa.br e seguir
o
link biblioteca. Lá vc poderá procurar dentre os títulos do Impa - a melhor
biblioteca em pós-graduação do RJ. A segunda melhor é a da UFRJ. Seu acervo
pode
ser consultado no site www.minerva.ufrj.br e procurar no acervo geral.


>
> o livro Porblemas Selecionados de Matemática - do Agostino eu tenho o
vol. 1
> (Fundamentos de álgebra e análise) - ao todo são quantos volumes ? de
que se
> tratam esses volumes ? será que é complicado consegui-los ? mesmo xerox


Se você estiver falando do livro de Raul Agostino, ele já foi comentado
aqui
na lista. Vale lembrar que o autor comentou comigo que NÃO pretende dar
continuidade ao projeto. Consulte o arquivo (da lista) para maiores detalhes.

[]'s

Alexandre Tessarollo

PS: Aqueles que quiserem, tenho um cópia xerox que tirei do exemplar que
o Raul
tem (ele só tem essa cópia, mais nenhuma). Quem quiser, só entrar em contato
que
dá-se um jetio de tirar um cópia dessa minha cópia. Em respeito aos outros
participantes da lista, peço que usem meu e-mail pessoal [EMAIL PROTECTED]




=
Instruções para entrar na lista, sair da lista e usar a lista em
http://www.mat.puc-rio.br/~nicolau/olimp/obm-l.html
O administrador desta lista é <[EMAIL PROTECTED]>
=



Re: [obm-l] Espirais da Montanha Russa

2002-01-31 Por tôpico Alexandre Tessarollo

"Rogério da Silva Ignácio" wrote:

  prezados participantes da lista.

  Preciso plotar uma simulação de uma montanha russa em 2D que tenha
pelo menos
  dois loopings.

  Pesquisei sobre as espirais usadas nos loopings das montanhas russas e

  encontrei referência a dois tipos:

  i  Em forma de parafuso (complicado para potar em 2D).
  ii Uma espiral em forma de gota (razão de ser desta mensagem).

  Bom, pesquisei o quanto pude sobre essa espiral a fim de saber seu
nome e sua
  parametrização. Encontrei apenas uma referência na revista
superinteressante de
  janeiro de 89. Nesta revista esplica-se que não se utiliza o formato
circular
  para o looping por razões físicas e biológicas pois permite pontos de
intensa
  gravidade e outros de nula. Então, segundo a revista, a solução foi
encontrada
  em 1977 numa curva chamada Espiral de cornu ou clotóide atribuída a
Euler.
  Porém não vejo nesta curva ( de paramétricas dy/dt=sen(t^2)
dx/dt=cos(t^2) )
  solução para o meu problema pois preciso plotá-la para ser vista "de
lado" e
  não "de frente" e o software que tenho que usar
  (http://phoenix.sce.fct.unl.pt/modellus) não possui recursos 3D.

  Alguém conhece alguma espiral que me sirva para esse fim? Ou como
plotar a
  clotóide em outra perspectiva?
  Agradeço antecipadamente
  Rogério

=

  Instruções para entrar na lista, sair da lista e usar a lista em
  http://www.mat.puc-rio.br/~nicolau/olimp/obm-l.html
  O administrador desta lista é <[EMAIL PROTECTED]>

=




Hum... Bem, resolvi "partir para ignorância" e pedi pro Maple
integrar as paramétricas que vc mostrou e depois
plotei. Como eu trabalho com curvas, reconheci a figura e consegui achar

a referência que eu tinha: Enciclopédia
Mirador de 1995, no vocábulo "curvas". Acredite ou não, não encontrei
nenhum outro livro de curvas planas que
mostrasse essa e algumas outras curvas... Segue o parágrafo da clotóide:

"Clotóide ou espiral de Euler (ou espiral de Cornu ou espiral de
Fresnel). É a curva de equações paramétricas

x = (a/sqrt(2)) * int[0,t]((sen(t)/sqrt(t))dt)
y = (a/sqrt(2)) * int[0,t]((cos(t)/sqrt(t))dt)

possuindo dois pontos assintóticos, de coordenadas (a*sqrt(pi/8),
a*sqrt(pi/8)) e (-a*sqrt(pi/8), -a*sqrt(pi/8)). Foi
inicialmente estudada por Euler e se originou de problemas relacionados
com molas; mais tarde teve sua importância
aumentada devido aos trabalhos de Fresnel sobre difração da luz."

Creio que não é permitido mandar arquivos attachados (Nicolau, que
tal escrever essas regrinhas básicas no
rodapé?), então vou mandar o arquivo diretamente para vc - são só 4k.

[]'s

Alexandre Tessarollo

OBS: int[a,b]((f)dx)  é a integral da função f em relação à variável x
no intervalo de a a b.

OBS: Desculpem a aparente demora, tive problemas na configuração dos
programas de e-mail e a lista estava negando as minhas msgs... Creio que
agora eu consegui resolver...

=
Instruções para entrar na lista, sair da lista e usar a lista em
http://www.mat.puc-rio.br/~nicolau/olimp/obm-l.html
O administrador desta lista é <[EMAIL PROTECTED]>
=



Re: 2 questões...

2001-12-13 Por tôpico Alexandre Tessarollo



"henrique.vitorio" wrote:
> 
> 
> Oi,
> Saudações a todos,meu nome eh Henrique(sow de
> Recife) e sow novo nessa lista.Entaum..aí vaum umas
> questões que gostaria que me ajudassem.

Bem-vindo...

>   1- encontre todas soluções inteiras positivas de:
>7^(x) + 1 = 5^(z) + 3^(y) (nessa questão soh consegui
> mostrar que x,y e z têm que ser ímpar).
>   2-Sabe-se que os vértices de um triângulo pertencem à
> hipérbole xy=1.Prove que seu ortocentro também pertence
> a essa hipérbole.


Bem, sejam A(a,1/a), B(b,1/b) e C(c,1/c) os vértices do triângulo e
suas respectivas coordenadas. Os coeficientes angulares das retas AB e
AC são -1/ab e -1/ac respectivamente. Assim, as alturas relativas a
esses lados terão coeficintes angulares ab e ac, respectivamente.
Completando as equações dessas alturas, ficam, respectivamente:
y = abx + 1/c - abc
y = acx + 1/b - abc

Para achar o ortocentro, basta resolver o sistema. encontraremos
x=-1/abc e y=-abc, logo, xy=1, CQD...

Sei que não expliquei tão bem quanto deveria, mas tenho o péssimo
hábito de checar e responder e-mails de madrugada. De qualquer forma,
usei o fato de que os três vértices estavam sobre a hipérbole xy=1
quando disse que A era (a, 1/a), por exemplo. Depois tirei a equação de
dois lados, das perpendiculares a esses lados passando pelo vértice
oposto e fiquei com duas equações. Como sabia que o único ponto em comum
era o ortocentro, bastou resolver o sistema e pronto. Para concluir,
bastou notar que, para quaisquer A, B e C na hipérbole xy=1, o
ortocentro de ABC também estará nessa hipérbole, C.Q.D.

> 
>  Desde jah agradeçoFalow!
>      Henrique

[]'s

Alexandre Tessarollo



Re: somatorio

2001-12-04 Por tôpico Alexandre Tessarollo



Gustavo Nunes Martins wrote:
> 
> Ha uma formula que diz que o a soma dos numero de uma PA que comeca com
> o numero n1 e acaba com o numero nx e
> (n1+nx)x/2. Um jeito muito simples de se descobrir esta formula pode ser
> econtrado em http://galileu.globo.com/edic/112/eureca.htm.
> 

Aliás, esse é o único jeito q conheço de DEDUZIR esta fórmula. Até é
possível PROVÁ-LA com indução, mas deduzir acho q só assim mesmo

> Como acho a expressao que me da a soma dos numeros da sequencia
> n^2 + (n+1)^2 + (n+2)^2 + (n+3)^2 + ... ?

Hum... Vc está querendo somar vários quadrados perfeitos consecutivos a
partir de um n^2 qualquer, certo? Pois bem, vamos passo a passo pq já
são passa das duas e meus neurônios foram dormir :-)

n^2 + (n+1)^2 + (n+2)^2 + (n+3)^2 + ... + (n+k)^2 =
n^2 + (n^2+2n+1) + (n^2+4n+4) + (n^2+6n+9) + ... + (n^2+2kn+k^2) =

Olhando com carinho, vemos um total de k+1 termos "n^2". Note também
que ops termos de primeiros grau podem ser arrumados de forma sugestiva
coma a abaixo. Por fim, restam os termos independentes. Vejamos como
fica...

(k+1)n^2 + (2+4+6+...+2k)n + (1+4+9+...k^2)

Fazendo por partes:
I) (k+1)n^2
É, acho q melhor q isso não fica, hehehe :-)

II)(2+4+6+...+2k)n
é uma PA. Assim, fica
k(k+1)n/2

III)(1+4+9+...k^2)
é a soma de todos os quadrados perfeitos a partir do primeiro (zero ou
um, tanto faz..). Se vc preferir, um caso particular do seu prob.
Observando com cuidado , temos:

4-1=3
9-4=5   5-3=2
16-9=7  7-5=2
25-16=9 9-7=2
.
.
.
Note que os quadrados não formam a PA q vc conhece, mas a diferença
deles sim. Isso significa q os quadrados pefeitos formam uma "PA de
segunda ordem", pq só na "segunda" vez q calculamos a diferença é q
chegamos a valores iguais. A PA q vc conhece é chamada de "PA de
primeira ordem", porque já na "primeira" vez q calculamos a diferença
chegamos a valores iguais. Podemos ter PAs de qualquer ordem

Mas, voltando ao nosso problema, temos q descobrir como somar essa tal
de "PA de segunda ordem". Como já são duas e meia, deixo p/vc tentar um
pouco. Sugiro tentar estabelecer alguma fórmula de recorrência, depois
uma do termo geral e, por fim, a da soma. Vale lembra q essas fórmulas,
na PA, são, respectivamente:
a[n] = a[n-1] + R
a[n] = a[1] + (n-1)R
S[n]=(a[1]+a[n])n/2

OBS: a[n] indica "n-ésimo termo da seqüencia"

Depois q você souber somar esta PA de segunda ordem basta somar I, II e
III q vc terá seu resultado... Se eu conseguir tempo amanhã eu faço o
resto...

[]'s

Alexandre Tessarollo



Re: Um quadrado repartido

2001-11-28 Por tôpico Alexandre Tessarollo



Ricardo Miranda wrote:
> 
> Olá. Alguem pode me ajudar a encontrar a área do triângulo formado
> interno ao quadrado na figura anexa?
> E quanto à medida das semi-retas que cortam o quadrado (as bases
> menores dos trapézios formados, que sao iguais aos lados do triângulo)?
> 
> Agradeço a ajuda,
> 
>  Ricardo Miranda M
> [EMAIL PROTECTED]
> 

Hum, na sua figura eu vou começar chamando de A o vértice superior
esquerdo, B C e D os vértices seguintes. M é o ponto médio de AB e P é o
ponto dentro de ABCD. Fazendo Pitágoras em AMD e BMC temos q
MD=MC=sqrt(5)/2. Prolongue MP até o lado CD e determine um ponto Q em
CD. Olhando para o triângulo retângulo MQC temo MQ=1, QC=1/2 e
MC=sqrt(5)/2. Logo, cos(QMC)=2/sqrt(5).  Seja MP=PC=x. Aplicando lei dos
cossenos em M no triângulo MPC, temos x=5/8, se eu não errei conta
alguma.

Agora, com tudo isso na mão, fica fácil. A segunda questão já foi
respondida, é x=5/8. A primeira, basta aplicar Heron ou o seu método
preferido e pronto: área de PCD = 3sqrt(2)/32.

Afora ewventuais erros de conta, acredito q tudo esteja certo...
Comentem...

[]'s

Alexandre Tessarollo



Re: Não deveria existir multiplicação por 0

2001-11-28 Por tôpico Alexandre Tessarollo



Wassermam wrote:

> queria saber pq a definição de algo infinito é dado por 0/0, que todos falam que é
> um 8 deitado e cortado.

Hum... Observe a equação 0x=0. Qualquer x que colocarmos servirá, certo? Pois
então, tente isolar x. Vc fica com x=0/0. Lembra que QUALQUER x servia? Pois é, o 0/0
existe, só que não conseguimos DETERMINAR quanto é 0/0 (o nosso x). Daí dizermos que
"0/0 é uma INDETERMINAÇÃO".

Quanto ao infinito, vc devia estar se referindo a função y=1/x para x=0. Podemos
até generalizar para y=k/x, k não nulo. Mas atenhamo-nos ao y=1/x por questões de
praticidade. Vc deve saber que o gráfico desta função é uma hipérbole com os eixos
cartesianos como assíntotas. Observando o gráfico dela, é fácil ver o que vou falar a
seguir. Se vc já tem alguma intimidade com a função y=1/x, pule os 2 próximos
parágrafos.

Bem, se vc não conhece recomendo que procure saber um pouco mais sobre esta
função, ela é muito interessante. A parte que nos cabe agora é o valor que ela assume
para x=0. Bem, vamos fazer x "ir" para 0 pela direita, ou seja, vamos pegar valores
POSITIVOS de x, cada vez menores, até chegar BEM perto de zero para ter uma idéia de
como a função se comporta. Para x=0,1, y=10. x=0,0001, y=1. Para
x=0,01, y = 1000...000. Olhando com carinho, vemos que, quanto menor o x,
maior o y. Ou seja, "quando x tende a zero pela direita, y tende a infinito".

Se fizermos de maneira análoga, só que pela esquerda. Ou seja, tomando valores
NEGATIVOS de x cada vez mais próximos de zero, veremos que acontece o mesmo que no
caso anterior, só que os valores de y são todos negativos. Adaptando a conclusão
anterior, temos que "quando x tende a zero pela esquerda, y tende a "menos" infinito".
Ou seja, quando x for zero, y teria um valor infinito, mas não sabemos se com sinal
positivo ou negativo.

Por isso dizemos que, quando x tende a zero, 1/x tende a infinito.

Quanto ao oito deitado e cortado, bem, você deve estar falando do símbolo de
infinito. Eu particularmente só conheço como oito deitado mesmo, não me lmbro deste
"corte" q vc está falando. Mas trata-se apenas de um símbolo, um desenho escolhido
para representar um idéia, assim como uma "bolinha vazia" foi escolhida para
representar a idéia de "nada", ou seja, o zero.

Espero que tenha ajudado. :-)

[]'s

Alexandre Tessarollo




Re: 2 de geometria

2001-11-27 Por tôpico Alexandre Tessarollo

Aqueles que quiserem uma figurinha, podem me pedir, eu tenho aqui. Pela
recente mesngame do volume de óleo já vi q é possível mandar anexos, mas
antes gostaria de saber qual a política oficial da lista com respeito a
anexos. Nicolau, por favor...

Alexandre Tessarollo wrote:
> 
> Ao povo q gosta de gemetria, seguem duas. A primeira foi um aluno que
> me passou, tirada de um livro de segundo grau das antigas ("Exercícios
> de Gemetria Plana", do Edgar de Alencar Filho). A outra acho q até já
> postei aqui, não tenho certeza. Mas tirei de outra lista. Vamos a elas:
> 
> 1) Por um ponto P interno a um triângulo ABC traçam-se a rretas
> paralelas aos seus lados, que o decompõem em seis partes, três das quais
> triângulos de área S[1], S[2] e S[3]. Achar a área do triângulo ABC.
> 
> Eu até tenho a respoata, mas ainda não sei como chegar nela.
> 
> 2) Tome um pentágomno qualquer ABCDE e as retas suportes dos lados. Note
> que elas detrminam um triângulo em cima de cada lado do pentágono.
> Construa as circunferências circunscritas a esse triângulos. Note que as
> circunferências de lados adjacentes se interceptam duas a duas em dois
> pontos: um dos vértices do pentágono e outro. Chamemos esses "outros"
> pontos convenietemente de A', B', C', D' e E'. Prove que A', B', C', D'
> e E' pertencem a uma mesma circunferência.
> 
> Não lembro de qual das n! listas que tirei esta questão, mas lembro q
> mencionava uma relação não confirmada a algum político chinês recente.
> Não sei se teria sido este chinês a formular ou se foi devido a ele que
> o prob ficou conhecido. Seria uma caso similar ao "problema do cavalo do
> presidente", né Nicolau? :0)))
> 
> []'s
> 
> Alexandre Tessarollo



2 de geometria

2001-11-26 Por tôpico Alexandre Tessarollo


Ao povo q gosta de gemetria, seguem duas. A primeira foi um aluno que
me passou, tirada de um livro de segundo grau das antigas ("Exercícios
de Gemetria Plana", do Edgar de Alencar Filho). A outra acho q até já
postei aqui, não tenho certeza. Mas tirei de outra lista. Vamos a elas:

1) Por um ponto P interno a um triângulo ABC traçam-se a rretas
paralelas aos seus lados, que o decompõem em seis partes, três das quais
triângulos de área S[1], S[2] e S[3]. Achar a área do triângulo ABC.

Eu até tenho a respoata, mas ainda não sei como chegar nela.

2) Tome um pentágomno qualquer ABCDE e as retas suportes dos lados. Note
que elas detrminam um triângulo em cima de cada lado do pentágono.
Construa as circunferências circunscritas a esse triângulos. Note que as
circunferências de lados adjacentes se interceptam duas a duas em dois
pontos: um dos vértices do pentágono e outro. Chamemos esses "outros"
pontos convenietemente de A', B', C', D' e E'. Prove que A', B', C', D'
e E' pertencem a uma mesma circunferência.

Não lembro de qual das n! listas que tirei esta questão, mas lembro q
mencionava uma relação não confirmada a algum político chinês recente.
Não sei se teria sido este chinês a formular ou se foi devido a ele que
o prob ficou conhecido. Seria uma caso similar ao "problema do cavalo do
presidente", né Nicolau? :0)))

[]'s

Alexandre Tessarollo



Re: Reformulando um problema mal definido

2001-11-22 Por tôpico Alexandre Tessarollo



Gustavo Nunes Martins wrote:
> 
> Caiu uma pergunta num vestibular e desconfio que ela esteja
> mal-formulada. Vejam:
> 
>  =< significa "menor que ou igual a" e k^y significa "k elevado a y".
> 
> Questao:
> 
> Fato 1: Sabe-se que cos(x) = (n-2)/3
> Fato 2: E sabido que a= Calcule a+b
> -Fim da questao-
> 
> Como os valores de cos(x) so podem estar entre -1 (inclusive) e +1
> (inclusive), 'n' pode ser qualquer coisa entre -1 (inclusive) e +5
> (inclusive). Essa conclusao sera chamada de conclusao 1.
> 
> Nada do que foi escrito no enunciado impede que 'b' seja, por exemplo,
> 10^727, pois esse valor nunca contraria o fato 2, que e o fato de que so
> e sabido que 'b' e um numero qualquer maior ou igual a 'n'.
> 
> Tambem pelo fato 2 e pela conclusao 1, o numero 'a' pode ser -10^747,
> pois e menor que qualquer valor possivel de 'n'. 'a' ainda pode ser
> -10^767 e muitos outros valores.
> 
> Concluo que a+b nao tem um valor fixo.
> 
> Acho a questao mal-feita. Quem a formulou nao perguntou o que desejava
> perguntar: ache a soma do menor valor possivel de 'n' com o maior valor
> possivel de 'n'.
> 
> Conheco gente que "resolveu" essa questao que que o que foi informado
> era que 'a' era o minino valor possivel de 'n' e que 'b' era o maximo
> possivel.
> 
> Como formular bem esta questao ultizilando apenas simbolos matematicos?


Hum... A única maneira que me ocorre é:
a=MIN(n)
b=MAX(n)

Creio q esta seja a forma correta. No entanto, por ser uma questão de
um vestiba, não acredito ser a mais apropriada, afinal, os alunos de 2o
grau não conhecem esta notação. (Para ser sincero, não estou muito
seguro desta notação.) Não sei como faria isto só com notação
p/vestibulandos... Eu usaria texto mesmo...

[]'s

Alexandre Tessarollo



Re: Um tal de Newton...

2001-11-21 Por tôpico Alexandre Tessarollo



"Alexandre F. Terezan" wrote:
> 
> Uma pequena distracao:
> 
> (1 + 3x + 2x^2) = 2(x+1)(x+1/2)
> e nao
> (1 + 3x + 2x^2) = (x+1)(x+1/2)
> 


Razões pelas quais pessoas como eu não deveriam mandar e-mails às 2:40
da manhã... Não sei pq ainda insisto... hehehe

[]'s

Alexandre Tessarollo

> -Mensagem Original-
> De: "Alexandre Tessarollo" <[EMAIL PROTECTED]>
> Para: <[EMAIL PROTECTED]>
> Enviada em: Quarta-feira, 21 de Novembro de 2001 02:41 Terezan
> Assunto: Re: Um tal de Newton...
> 
> [EMAIL PROTECTED] wrote:
> >
> > Meus cumprimentos,
> >
> > Estava estudando "um tal de Newton" e encontrei uma questão
> > interessante, embora eu esteja errando algo simples pra vocês...
> >
> > Questão (FFCLUSP)
> > Mostrar que o coeficiente de x^8 no desenvolvimento
> > de (1 + 3x + 2x^2)^10 é 3780.
> >
> > Meu erro: os coeficientes de x e de x^2 estão fazendo o
> > coeficiente do termo x^8 ficar muito grande ...
> 
> Vale lembrar q se vc estiver tentando usar a fórmula do Binômio de
> Newton, ela só vale para BInômios, ou seja, algo como (x+a)^n. Nós temos
> um TRInômio...
> 
> Bem, mas vamos tentar... Sabemos q o trinômio pode ser reescrito como
> (x+1)(x+1/2). Assim, queremos saber o coeficiente de x^8 no
> desenvolvimento de
> [(x+1)(x+1/2)]^10= [(x+1)^10][(x+1/2)^10]
> 
> Seja a[i]x^i o termo de grau "i" do primeiro binômio e, p/não
> confunidir as letras, a[j]x^j o de grau "j" do segundo binômio. Assim, o
> nosso polinômio final terá termos da forma a[i]a[j]x^(i+j), com "i" e
> "j" variando (independentemente) de 0 a 10.
> 
> Dessa forma, temos que achar i+j=8. As soluções (i;j) que estão no
> nosso intervalo são: (0;8), (1;7), (2;6), (3;5), (4;4), (5;3), (6;2),
> (7;1) e (8;0). Agora sim podemos utilizar a fórmula do Binômio de
> Newton, calcular os coeficientes com "i" e "j" das soluções, fazer as
> devidas contas e pronto. Sei q deve dar algum trabalho, mas depois posso
> até fazer caso alguém queira. Como a essa hora meus neurônios já foram
> dormir, fico devendo uma solução mais concisa e prática.
> 
> []'s
> 
> Alexandre Tessarollo
> >
> > Caso alguém queira tentar...
> >
> > Muito grato,
> >
> > Héduin Ravell
> >
> > _
> > Do You Yahoo!?
> > Get your free @yahoo.com address at http://mail.yahoo.com



Re: Somatórios

2001-11-21 Por tôpico Alexandre Tessarollo



Bruno Furlan wrote:
> 
> Desculpe, erro crasso... mas a série harmônica diverge (aliás, só por
> curiosidade, como se prova isso?)

Uma maneira é separá-la assim:
1+
1/2+
1/3+1/4+
1/5+1/6+1/7+1/8+
...
1/(2^n+1)+...+1/[2^(n+1)]+
...

Note que a soma de cad linha é sempre maior que 1/2, exceto,
obviamente, na segunda. Somando a segunda e a terceira linha, dá um
número maior q um. Somando a qurta e a quinta, dá outro número maior que
um. E assim por diante. No final, teremos vários números maiores que um.
Na verdade, teremos infinitos números maiores que um. Não conseguimos
somar tudo porque o resultado seria "infinito". Logo, a série diverge.

Vale lembrar q esta NÃO é uma prova completa, pois não demonstrei que a
soma de 1/2^n+...+1/2^(n+1) é sempre maior que 1/2. 

Até onde me lembro, o argumento que prova isot é algo como "note que de
1/2^n até  1/2^(n+1) existem exatamente 2^n termos, TODOS maiores ou
iguais a 1/2^(n+1). Portanto, a soma desses termos será maior ou igual a
(2^n)*[1/2^(n+1)]=1/2.". Ei, acho q é este argumento mesmo, legal,
consegui lembrar... Alguém se habilita a ratificar ou retificar meu
argumento?

>, então como fazer para calcular o que foi
> pedido? O enunciado está errado?

Se o enunciado está corrteo, no lado direito da igualdade vc terá algo
da forma (x^3+1)*"infinito". (Não lembro exatamente de qual era o
polinômio, mas isso é irrelevante). Produto de um número por "infinito"
dá "infinito". Ou seja, não há resultado válido e vc não consegue
resolver o prob. O enunciado está errado ou realmente não há resposta.

Alguém chegou a cogitar a hipótese de ser Somatório de (1/2^n), com n
variando no mesmo intervalo. Isso seria uma PG e teria como solução a
que já foi apresentada aqui na lista. Sei que deveria citar os nomes,
mas minha memória anda péssima e estou meio cansado p/procurar nos
e-mails antigos.. :-)

[]'s

Alexandre Tessarollo



Re: Um tal de Newton...

2001-11-20 Por tôpico Alexandre Tessarollo



[EMAIL PROTECTED] wrote:
> 
> Meus cumprimentos,
> 
> Estava estudando "um tal de Newton" e encontrei uma questão
> interessante, embora eu esteja errando algo simples pra vocês...
> 
> Questão (FFCLUSP)
> Mostrar que o coeficiente de x^8 no desenvolvimento
> de (1 + 3x + 2x^2)^10 é 3780.
> 
> Meu erro: os coeficientes de x e de x^2 estão fazendo o
> coeficiente do termo x^8 ficar muito grande ...

Vale lembrar q se vc estiver tentando usar a fórmula do Binômio de
Newton, ela só vale para BInômios, ou seja, algo como (x+a)^n. Nós temos
um TRInômio...

Bem, mas vamos tentar... Sabemos q o trinômio pode ser reescrito como
(x+1)(x+1/2). Assim, queremos saber o coeficiente de x^8 no
desenvolvimento de
[(x+1)(x+1/2)]^10= [(x+1)^10][(x+1/2)^10]

Seja a[i]x^i o termo de grau "i" do primeiro binômio e, p/não
confunidir as letras, a[j]x^j o de grau "j" do segundo binômio. Assim, o
nosso polinômio final terá termos da forma a[i]a[j]x^(i+j), com "i" e
"j" variando (independentemente) de 0 a 10.

Dessa forma, temos que achar i+j=8. As soluções (i;j) que estão no
nosso intervalo são: (0;8), (1;7), (2;6), (3;5), (4;4), (5;3), (6;2),
(7;1) e (8;0). Agora sim podemos utilizar a fórmula do Binômio de
Newton, calcular os coeficientes com "i" e "j" das soluções, fazer as
devidas contas e pronto. Sei q deve dar algum trabalho, mas depois posso
até fazer caso alguém queira. Como a essa hora meus neurônios já foram
dormir, fico devendo uma solução mais concisa e prática.

[]'s

Alexandre Tessarollo
> 
> Caso alguém queira tentar...
> 
> Muito grato,
> 
> Héduin Ravell
> 
> _
> Do You Yahoo!?
> Get your free @yahoo.com address at http://mail.yahoo.com



Re: Somatórios

2001-11-20 Por tôpico Alexandre Tessarollo

Bruno Furlan

>Nunca estudei limite de soma, mas vou tentar:
> 
>Sum[(x³+13)/2n] = (x³+13).Sum(1/2n) = (x³+13)/2.Sum(1/n).
>Como o limite de Sum(1/n) é 2, temos que Sum[(x³+13)/2n] = x³+13.

Concordo com tudo antes e tudo depois dessa linha. Só não concordo quan
vc diz que o limite da soma de 1/n com n variando de 1 a infinito dá 2.
A soma da PG (1/2)^n com n variando de zero (só p/somar 1) a infinito é
que dá 2. O somatório conforme apresentado representa a série harmônica
- a primeira que se estuda quando se aprende séries "de verdade". Creio
inclusive que este tema já foi discutido aqui... Se alguém se interessar
há uma "prova" bem intuitiva e convincente.

[]'s

Alexandre Tessarollo



Re: RES: IME (era: "Re:dúvida")

2001-11-14 Por tôpico Alexandre Tessarollo



"M. A. A. Cohen" wrote:

> Oi Alexandre! Bom, acabei de comentar essa questao :)) Acho que o pessoal
> nao comentou pq a lista estava com problemas!

Oops! Esqueci deste pequeno detalhe... Minhas desculpas a quem por ventura
se sentiu ofendido de alguma forma...

>
> Quanto as suas observacoes, cabem alguns comentarios!
> A primeira solucao que vc coloca (divulgada pelo gpi) eh essencialmente
> identica a ´solucao do Poliedro´ (trocando y por f(x)).

De fato é. Só que o Poliedro faz o trabalho sujo e o GPI pura e simplesmente
"joga" um "é fácil ver" e pronto.

> A maioria dos alunos
> de ensino medio sabem ateh mais do que o que vc disse. Sabem que se vc tem
> uma bijecao de A em ACR e vc coloca os dois graficos num mesmo eixo, entao
> eles sao simetricos em relacao a bissetriz y=x e em particular soh podem se
> encontrar sobre a reta.

Hum... Isso validaria a solução do GPI, mas eu honestamente não me lembro de
ter visto isto em nenhum livro de segundo grau. Se vc puder dar referências,
agradeço...

>
> Soh nao concordo que a segunda solucao esteja totalmente correta, embora a
> ideia seja legal.
> Nao me parece obvio, a principio, que a sequencia f(f(f( .. .f(x) ))) seja
> convergente (talvez ateh seja e eu nao esteja vendo o motivo direto). De
> fato, eh quase que imediato do enunciado que se a sequencia tiver sempre um
> numero par de f´s, entao ela converge (de fato eh constante qdo x satisfaz a
> equacao dada). Se tem um numero impar de f´s, ela tmb converge (tmb eh
> constante). Falta mostrar que esses dois valores sao iguais, e isso eh mto
> parecido com resolver o problema inicial.. Uma solucao feita desse jeito
> esta eh www.pensi.com.br

Hum... É, o Pensi indicou o caminho bem. De fato a segunda solução (do
Planck) poderia ser mais clara...

>
> Vc vai inclusive notar varias semelhancas entre essa e a sua penultima
> solucao, pq essencialmente o problema final eh o mesmo.
> A sua ultima solucao eh bastante legal. Essa foi a solucao do curso Elite,
> que eu falei no ultimo email. Nao conheco esse professor que vc cita, mas se
> nao me engano, essa solucao tambem foi dada em prova, por um aluno do elite.

Esse foi um professor meu no segundo grau. Tb escreveu um livro de problemas
que recentemente foi mencionado aqui...

>
> t+
> Marcio

[]'s

Alexandre Tessarollo

>
>
> -Mensagem original-
> De: [EMAIL PROTECTED]
> [mailto:[EMAIL PROTECTED]]Em nome de Alexandre
> Tessarollo
> Enviada em: quarta-feira, 14 de novembro de 2001 18:18
> Para: [EMAIL PROTECTED]
> Assunto: IME (era: "Re:dúvida")
>
> luis felipe wrote:
>
> > concordo com o alexandre
> >
> > a prova do IME deste ano foi bem elaborada, embora eu ache que duas
> questões
> > estavem pesadas demais para alunos de 2 grau( 7 e a 9) devemos lamentar
> > também uma falha grave no enunciado da questão 8
> >
> > valeu
> >
> > luis felipe
>
> Já que ninguém comenta, comento eu. Comecemos pela questão 9.
>
> "Resolva a equação sqrt(5-sqrt(5-x))=x sabendo-se que x>0."
>
> Eu já devo ter visto umas 4 soluções diferentes, mas em quase todas
> havia
> pelo menos um passo não justificado ou "questionável"... Uma delas era:
>
> Seja f(x) = sqrt(5-x). Temos f(f(x))=x. Logo,  f(x)=f^(-1)(x). Aí vem a
> parte "é
> fácil ver que" os gráficos de f(x) e de f^(-1)(x) se cruzam sobre a reta
> y=x. A
> partir daí, temos f(x)=x, resolve-se uma equação do segundo grau e pronto.
> Mas
> falta demonstrar a parte "é fácil ver"...
>
> Outra diz:
>
> Aplicando f(x) nela mesma 2n vezes, com n tendendo ao infinito, teremos
> f(f(f(f((f(x))...=x. Logo, podemos trocar todos os
> f(f(f...(f(x))...)))
> de "dentro" do primeiro "f" por "x". Assim teremos f(x)=x e novamente é só
> resolver a eq do segundo grau. A solução, olhando com carinho, está certa,
> mas
> foi utilizado o conceito de limite.
>
> Ainda há uma terceira, esta já sem erros mas um pouco mais longa. Trata-se
> da
> solução do Poliedro:
>
> Como x>0 e real, temos que 0 original
> transforma-se em
> sqrt(5-y)=x (II)
> Elevando I e II ao quadrado, temos:
> y^2=5-xIII
> x^2=5-yIV
> Fazendo III-IV, temos
> y^2-x^2=y-x
> (y+x)(y-x)=y-x
> (y+x)(y-x)-(y-x)=0
> (y-x)(y+x-1)=0
>
> Segue que
> y-x=0V
>
> OU
>
> y+x-1=0VI
>
> De V segue a nossa equação do segundo grau. Considerando o intervalo
> 0 só teremos uma resposta - a certa. Falta examinar VI. Substituindo-a em III
> ou
> IV, teremos uma equação do segundo grau que resulta só uma resposta no
> intervalo
> 0 IV,
&g

IME (era: "Re:dúvida")

2001-11-14 Por tôpico Alexandre Tessarollo



luis felipe wrote:

> concordo com o alexandre
>
> a prova do IME deste ano foi bem elaborada, embora eu ache que duas questões
> estavem pesadas demais para alunos de 2 grau( 7 e a 9) devemos lamentar
> também uma falha grave no enunciado da questão 8
>
> valeu
>
> luis felipe

Já que ninguém comenta, comento eu. Comecemos pela questão 9.

"Resolva a equação sqrt(5-sqrt(5-x))=x sabendo-se que x>0."

Eu já devo ter visto umas 4 soluções diferentes, mas em quase todas havia
pelo menos um passo não justificado ou "questionável"... Uma delas era:

Seja f(x) = sqrt(5-x). Temos f(f(x))=x. Logo,  f(x)=f^(-1)(x). Aí vem a parte "é
fácil ver que" os gráficos de f(x) e de f^(-1)(x) se cruzam sobre a reta y=x. A
partir daí, temos f(x)=x, resolve-se uma equação do segundo grau e pronto. Mas
falta demonstrar a parte "é fácil ver"...

Outra diz:

Aplicando f(x) nela mesma 2n vezes, com n tendendo ao infinito, teremos
f(f(f(f((f(x))...=x. Logo, podemos trocar todos os f(f(f...(f(x))...)))
de "dentro" do primeiro "f" por "x". Assim teremos f(x)=x e novamente é só
resolver a eq do segundo grau. A solução, olhando com carinho, está certa, mas
foi utilizado o conceito de limite.

Ainda há uma terceira, esta já sem erros mas um pouco mais longa. Trata-se da
solução do Poliedro:

Como x>0 e real, temos que 0


Re: dúvida

2001-11-14 Por tôpico Alexandre Tessarollo



Arnaldo wrote:

> >
> >Um automóvel comporta dois passageiros nos bancos da frente e três no de>trás.
>
> >Calcule o número de alternativas distintas para lotar o automóvel com pe>ssoas
>
> >escolhidas dentre sete, de modo que uma dessas pessoas nunca ocupe um lugar
>
> >nos bancos da frente
> >
> >Vamos chamar de (A) a pessoa que nunca pode ocupar o banco da frente. Quando
> (A) ocupa um dos bancos de trás sobram 6 pessoas para ocupar os outros quatro
> bancos e portanto temos arranjo(6,4) = 360 maneiras distintas,mas como (A) pode
> acupar três bancos então o total de maneiras distintas são 3*360 = 1080.
>

Hum... Praticamente correto. Se o cidadão estiver no carro, então realmente
teremos 1080 maneirs de arrumar todo mundo. Mas ele pode ficar de fora, por quê
não? Nesse caso, teríamos 6 pessoas para distribuir em 5 lugares. Fazendo as
contas, temos 720 maneiras. Somando os dois casos (com A e sem A), temos
1080+720=1800.

[]'s

Alexandre Tessarollo

PS: O povo daqui não vai comentar a prova do IME deste ano não? Estava
interessantíssima, especialmente se tentarmos resolvê-la APENAS com conteúdo de 2º
grau/Ensino Médio...




Re: RES: ajuda em um problema (oops)

2001-10-11 Por tôpico Alexandre Tessarollo


[...]

...esse quadrilátero [MNPQ] é _circunscritível_ e I é seu centro, já que
ele equidista dos lados... 

Oops!!! Minhas mais sinceras desculpas!! Pessoas como eu não deviam ler
nem tampouco responder e-mails de amdrugada... Tinha lido "incritível"
ao invés de "circunscritível"... Minhas mais sinceras desculpas ao
Einstein e ao Harold... Tanto a questão como a resposta estão
corretíssimas, e isso eu digo agora às 15:20h, em sã consciência!! :-))

Bem, de qualquer forma aproveito para lançar outra questão: Seja ABCDE
um pentágono QUALQUER. Prolongue os lados de forma que eles se encontrem
dois a dois, determinando os triângulos FAB, GBC, HCD, IDE, JEA sendo F,
G, H, I e J os pontos de intersecção (Agora a figura tornou-se um
pentágono estrelado). Construa os círculos circunscritos a estes
triâgulos e note que eles se interceptam em dois pontos dois a dois.
Cinco pontos são A, B, C, D, e E e os outros cinco chamemos
convenientemente de A', B', C', D' e E'. Prove que A'B'C'D'E' é
circuncritível para qualquer ABCDE (convexo?).

[]'s

Alexandre Tessarollo



Re: RES: ajuda em um problema

2001-10-08 Por tôpico Alexandre Tessarollo

Solução:
Faz a figura para ficar mais fácil de ver...
Como M, N, P e Q são as projeções e I  sobre  os lados AB, BC, CD, DA
temos que:
Os quadriláteros BMNI, NIPC, PIQD, MIQA são todos inscritíveis já que
possuem angulos opostos somando 180 graus. 
Como o quadrilátero ABCD é inscritível, temos que: 


Re: RES: ajuda em um problema

2001-10-08 Por tôpico Alexandre Tessarollo

Solução:
Faz a figura para ficar mais fácil de ver...
Como M, N, P e Q são as projeções e I  sobre  os lados AB, BC, CD, DA
temos que:
Os quadriláteros BMNI, NIPC, PIQD, MIQA são todos inscritíveis já que
possuem angulos opostos somando 180 graus. 
Como o quadrilátero ABCD é inscritível, temos que: 


Re: ajuda em um problema e Extra

2001-10-08 Por tôpico Alexandre Tessarollo



> harold wrote:
> 
> seja ABCD um quadrilátero convexo inscrito num círculo e seja I  ponto
> de intersecção das suas diagonais. As projeções de sobre os lados AB,
> BC,CDe DA
> são respectivamente ,M,N,P e Q. Prove que o quadrilátero MNPQ é
> circunscrítivel a um círculo com centro em I.

Projeções de QUEM sobre os lados? Tentei projetar I e a afirmação
tornou-se falsa (leia-se achei diversos contra-exemplos). Fiz o mesmo
com O e tampouco funcionou... Seja claro e específico, plz.

[]'s

Alexandre Tessarollo

PS: Aproveitando a deixa, passo mais um:

Sabendo que sen 2A, sen 2B e sen 2C estão em PA nessa ordem, demonstrar
que tan (B+C), tan (C+A) e tan (A+B) também estão em PA nessa ordem.



Re: Respostas CN

2001-10-03 Por tôpico Alexandre Tessarollo


Raul Agostino foi um dos meus professores no segundo grau e hoje em dia
é meu colega. Já conversei com ele e ele disse que não pretende dar
continuidade a esse livro - que é, na verdade, a parte um. Também disse
que não deve ser reeditado, portanto esqueça livrarias normais e comece
a procurar em sebos ou arranje uma cópia com alguém.

[]'s

Alexandre Tessarollo

PS: Vai dar trabalho conseguir uma cópia, mas garanto que valhe a pena. 
Quanto a ser usado no vestiba da rs, bem, isso eu não sei...

> Igor Castro wrote:
> 
> Gostaria de te indicar o PROBLEMAS SELECIONADOS DE MATEMÁTICA -
> Antônio Luis Santos e Raul F. W. Agostinho, mas vai ser dificil você
> achar... Esse foi o que eu mais usei para passar e comparando com
> morgado e etc, este é muito melhor e ainda...rs... todo ano cai uma
> questão desse livro, desde 99... nem sei se podia dizer isto...
> Igor..
> []'s



Re: opf

2001-09-29 Por tôpico Alexandre Tessarollo



pichurin pichurin wrote:
> 
>  --- romenro <[EMAIL PROTECTED]> escreveu: > Eu sou
> de São José dos Campos, estou no 2º colegial,
> >
> > achei a prova facil, com excessão da questão 16 e
> > 17. Vc
> > sabe resolve-las?
> >
> >
> >
> __
> > AcessoBOL, só R$ 9,90! O menor preço do mercado!
> > Assine já! http://www.bol.com.br/acessobol
> 
> sim, consegui resolvê-las mas acabei errandoa 16
> mas na 17 posso te ajudar:
>  QV1=qV2
> ((M(V1)^2)/2)= (m(V2)^2)/2
> 
> aí é só resolver o sistema assim
> (Q/q)=(V2/V1)
> ((v1)^2/(V2)^2)= M/m
> V2/V1=(M/m)^(1/2)
> 
> portanto:
> 
> (Q/q)=(M/m)^(1/2)
> >
> >
> 
> 

Pô... Será q vcs não podem indicar algum site com a prova? Ou pelo
menos  colocar os enunciados dessas duas (16 e 17) p/a gente discutir...

[]'s

Alexandre Tessarollo

PS: Quanto a ser off-topic, creio q discussões sobre Física são (como já
foram) sempre bem-vindas... Correto, Nicolau?



Re: Geometria

2001-08-31 Por tôpico Alexandre Tessarollo


 
Eduardo Casagrande Stabel wrote:
Ola pessoal!
Eu tenho duas perguntas sobre geometria.
1. Sejam A e B dois pontos distintos do plano, qual o nome, propriedade
ou
quaisquer coisa relacionada a figura formada por todos os pontos P
de tal
forma que o angulo A^PB seja constante?
 

    Ao conjunto L dos pontos do plano que têm
uma determinada propriedade P, tal que todo elemento de L possui a propriedade
P e apenas os elementos de L possuem a propriedade P dá-se o nome
de lugar geométrico ou, em inglês e latim, locus. Um
exemplo é a circunferência. Todos os pontos da circunferência
possuem a propriedade de distar R do centro O, e apenas os pontos da circunferência
possuem esta propriedade.
    O lugar geométrico que vc procura chama-se
Arco Capaz. Dado um ângulo XYZ e um segmento AB, constrói-se
assim:
1) transporte o ângulo XYZ para o segemento AB, ou seja, AB será
um dos lados do ângulo e A será o vértice. Se vc sabe
como fazer isto, pule para o passo 2.
    a) Com um distância qualquer no compasso, faça
um arco centrado em Y e que corte YX e YZ.
    b) Com esta MESMA distância, centre em A e
faça a circunferência. (Poderia ser só um arco que
cortasse AB e fosse um "pouquinho" maior que o arco original, mas daqui
a pouco vc vai entender o porquê da circunferência completa).
    c) Seja T e U as intersecções do arco
(construído no item a) com YX  e YZ, respectivamente.
    d) Marque a distância TU no compasso.
    e) Seja C a intersecção da circunferência
(construída no item b) com o segmento AB.
    f) Centrando em C, marque a distância (TU)
na circunferência (construída no item b). Surgirão
os pontos D ("acima" de AB) e E ("abaixo" de AB).
    g) Agora, temos BAD = BAE = XYZ
2) Tomemos o ângulo BAD = XYZ (D "acima" de AB). Construa a perpendicular
a AD que passa por A.
3) Construa a mediatriz de AB.
4) Seja O a intersecção da mediatriz de AB com a perpendicular
(construída no item 2).
5) Centrando em O e com o compasso "indo" até A, faça
o arco "até" B. Note que vc pode tanto fazer um arco grande como
um arco pequeno. Faça o arco "para baixo". (Se o seu ângulo
for agudo, será o arco maior. Se for obtuso, será o arco
menor.)
6) Repita os passos 2 a 5, sendo que no passo 2 você deverá
pegar o ângulo BAE ao invés de BAD (E "abaixo" de AB) e no
passo 5, você deverá construir o arco "para cima".
 
    A junção destes dois arcos dá
o Arco Capaz que enxerga AB sob um ângulo XYZ. A figura no
final parece um oito deformado se XYZ for agudo, uma quase-elipse achatada
se o ângulo for obtuso e é exatamente uma circunferência
se o ângulo for reto.
 
    Prova:
    Observe que o ângulo OAB é comlementar
de BAD = XYZ por construção. Seja M o ponto médio
de AB. O ângulo AMO é reto pois MO é mediatriz de AB,
por construção. Assim, como num triângulo a soma dos
ângulos internos é 180º, no triângulo AMO, o ângulo
AOM deverá ser igual a BAD = XYZ.
    Como AM=MB, MO=MO e os ângulos AMO=BMO, temos,
por lado-ângulo-lado, que os triângulos AMO e BMO são
congruentes e, em particular, os ângulos BOM=AOM=XYZ. Ou seja, o
ângulo AOB é 2*XYZ.
    Mas AOB é o ângulo central do arco (construído
em 5). Logo, qualquer ponto neste arco enxergará AB sob um angulo
de AOB/2 = (2*XYZ)/2 = XYZ.
    Repete-se a prova para o "outro" arco (construído
em 6) e está provado, CQD.
 
    Quaisquer dúvidas ou comentários,
estamos aí.
[]'s
Alexandre Tessarollo
PS: No passo 5, na hora de construir o arco "para cima" ou "para baixo",
basta lembrar que o arco tem de estar no lado "oposto" ao lado em que o
ângulo foi construído, com relação à
AB. Ou seja, se você construir BAD "acima" de AB, o arco deverá
ser "abaixo", e vice-versa.
 
2. Temos um triangulo equilatero ABC, dentro do triangulo se tracam
tres
segmentos, cada um partindo de um lado, que nao se cruzam mas estao
um
apoiados nos outros. Mostrar que se os tres segmentos tem o mesmo
comprimento, o triangulo do meio e' tambem equilatero.

Eu ia anexar uma figurinha pro 2, mas tem gente que
nao consegue abrir.
 
A

/\
   
/  \
  
/    \
 
/   \

/ \
   /
\
 /
\
    /   
x Q
   /  
\
 R  x  
\
    /  
\
   /  
\
  /  
\
/__x___\
B 
P 
C
 
    Hum, talvez mais atrapalhe do que ajude, mas vamos
ver se eu entendi: Sejam X, Y e Z as intersecções de AP e
BQ, BQ e CR, CR e AP, respectivamente. Então, se ABC é equilátero
e AX=BY=CZ, então o triângulo XYZ também é equilátero.
    Bem, se for isto, acho que até já vi
essa questão antes. Só que não sei se vi a solução
e, se vi, não lembro. Vou pensar um pouco mais... :-)
Eduardo Casagrande Stabel.

Mais uma vez,
[]'s
Alexandre Tessarollo


Re: eu estou tentando fazer essa questão não consigo, alguem me ajuda?

2001-08-16 Por tôpico Alexandre Tessarollo


Hum... Vamos ver se consigo.. Primeiro: Quantos conjuntos de quatro pneus
podemos formar? Como escolher quatro pneus para rodar equivale a deixar um na
mala, é fácil ver que temos 5 grupos distintos, sendo que o estepe participa de
4 (note que só no grupo original o estepe está na mala, nos outros ele está
rodando). Assim, temos que dividir nossa viagem de 2000km em 5 partes, uma
p/cada grupo. Como o estepe está em 4 dos cinco grupos e cada grupo roda
2000/5=400km, podemso dizer que o estepe roda 4*400=1600km. Correto?

[]'s

Alexandre Tessarollo

[EMAIL PROTECTED] wrote:

> Milton comprou um carro 0Km e , pensando em economizar os pneus, usou os
> quatro colocados mais o estepe, numa viagem cujo percurso foi de 2000Km. Se
> cada pneu rodou a mesma quilometragem, então o estepe foi usado nessa viagem
> por quantos quilômetros?




Re: En: Undelivarable: Re:

2001-07-24 Por tôpico Alexandre Tessarollo



 
    Ahhh, agora entendi o que vc quis dizer... Minha
msg (abaixo) chegou numa boa na lista, mas eu recebi o tal e-mail do mandic...
Bem, neste caso, só esperando uma resposta do nosso moderador mesmo...
Nicolau, por favor manifeste-se o quanto antes...
[]'s
Alexandre Tessarollo
Alexandre Tessarollo wrote:
 
    Hum, estou na lista já há um bom tempo,
mas acho q vc estava bem antes... Provavelmente, no início dos tempos
desta lista, ela era hospedada no servidor Mandic, ou, pelo menos, o e-mail
era @mandic. Experimente atualizar seu livro de endereços / address
book para [EMAIL PROTECTED]
    Creio que isto deva resolver seus problemas imediatos,
de qualquer forma, procure/espere uma resposta do Nicolau, responsável
pela lista. Com certeza ele saberá explicar muito melhor do que
eu ;-)
[]'s
Alexandre Tessarollo
Jose Paulo Carneiro wrote:
 Alguem sabe
me explicar o que significa esta mensagem que ja recebi mais de uma vez?JP
- Original Message -
From: [EMAIL PROTECTED]
To: Jose
Paulo CarneiroSent: Monday, July 23, 2001 9:11 PMSubject:
Undelivarable: Re:
 Caro internauta,
A mensagem enviada para [EMAIL PROTECTED]não
pode ser entregue porque o domínio @mandic.com.br
mudou para @osite.com.br.
Por favor reenviesua
mensagem conforme exemplo abaixo:
DE : @mandic.com.br
PARA : @osite.com.br
O SITE,
SEU LUGAR NA INTERNET
 
 
O SITE recomenda:
Acesse o Atendimento
On-Line, via Chat.
Você não
precisa mais ter 2 linhas telefônicas para conectar
e falar com
o Suporte Técnico.
Agora você
pode navegar e ser atendimento ao mesmo tempo.
Clique no endereço
abaixo:
<http://cadastro.osite.com.br/suaconta/chatosite.shtml>
 
 
 
 
 
 






Re: En: Undelivarable: Re:

2001-07-24 Por tôpico Alexandre Tessarollo



 
    Hum, estou na lista já há um bom tempo,
mas acho q vc estava bem antes... Provavelmente, no início dos tempos
desta lista, ela era hospedada no servidor Mandic, ou, pelo menos, o e-mail
era @mandic. Experimente atualizar seu livro de endereços / address
book para [EMAIL PROTECTED]
    Creio que isto deva resolver seus problemas imediatos,
de qualquer forma, procure/espere uma resposta do Nicolau, responsável
pela lista. Com certeza ele saberá explicar muito melhor do que
eu ;-)
[]'s
Alexandre Tessarollo
Jose Paulo Carneiro wrote:
 Alguem sabe
me explicar o que significa esta mensagem que ja recebi mais de uma vez?JP  
- Original Message -
From: [EMAIL PROTECTED]
To: Jose
Paulo CarneiroSent: Monday, July 23, 2001 9:11 PMSubject:
Undelivarable: Re:
 Caro internauta,
A mensagem enviada para [EMAIL PROTECTED]
não pode ser entregue porque o
domínio @mandic.com.br mudou
para @osite.com.br.
Por favor reenvie
sua mensagem conforme exemplo abaixo:
DE : @mandic.com.br
PARA : @osite.com.br
O SITE,
SEU LUGAR NA INTERNET
 
 
O SITE recomenda:
Acesse o Atendimento
On-Line, via Chat.
Você não
precisa mais ter 2 linhas telefônicas para conectar
e falar com
o Suporte Técnico.
Agora você
pode navegar e ser atendimento ao mesmo tempo.
Clique no endereço
abaixo:
<http://cadastro.osite.com.br/suaconta/chatosite.shtml>
 
 
 
 
 
 





Re: AJUDA!!! ONDE encontro?

2001-07-02 Por tôpico Alexandre Tessarollo


Com paciência, tempo e dedicação, em sebos. Afora isso, livros da MIR,
conforme discutido aqui anteriormente, só nos sites:

www.livifusp.com.br  - Livraria de Física da USP, tem alguns vários da
MIR.
urss.ru  - Editora russa que herdou MUITOS livros da MIR. Há
em inglês, espanhol, francês, russo..

Se a minha parca memória não está me traindo, alguém até citou uma
loja/livraria/sebo/sei-lá-o-que que vendia livros da MIR, mas era em
Sampa e eu sou carioca, então nem cheguei a anotar o endereço. Se vc
quiser, peça p/a lista ou dê uma checada nos arquivos da lista.

[]'s

Alexandre Tessarollo

> Marcos Eike wrote:
> 
> Pessaol onde eu encontro o livro fundamentos de física elementar da
> editora MIR, se eu não me engano os autores são 6 russos
> 
> Ats,
> Marcos Eike



Re: 0,8... + 0,1... = 1 ?!

2001-06-24 Por tôpico Alexandre Tessarollo


Uma maneira de vc mostrar que dois números REAIS são distintos é
mostrar que existe pelo menos um número entre eles. Caso contrário, são
o mesmo número.

Assim, por exemplo, 1 e 2 são distintos pq posso, por exemplo, ver que
1,7 está entre eles. Já 4 e 4 são o mesmo número pq não consigo achar
nehum número que esteja entre 4 e 4.

Com 0,9... e 1, é a mesma coisa que 4 e 4: não conseguimos achar
alguém entre eles., Logo, são o mesmo número.

[]'s

Alexandre Tessarollo

PS: Vc talvez esteja com uma dúvida que eu tive por muito tempo. Era uma
raciocínio em que eu fazia uma analogia dos reais com os inteiros para
matar o argumento que eu expliquei acima. (Não) Funcionava assim:

"Pegue os números 1, 2, 3 e 4, por exemplo. 2 e 4 são distintos pq há o
3 entre eles. 1 e 2, APESAR de não haver ninguém (inteiro) entre eles,
são distintos* e 1 é o número que vem IMEDIATAMENTE antes de 2. 

Analogamente, pegue os números 0,999... e 1. Eles são distintos* e
0,999... é o número IMEDIATAMENTE antes de 1. A diferença 1-0,999...
seria uma "infinitesimal**", o número IMEDIATAMENTE depois de zero."

*O problema que eu não percebia era como saber que os números eram
distintos.

**Esse era o nome que eu dava para tal valor, usando o bom-senso de
chamar algo muito pequeno (1-0,999...) por um nome que lembrasse algo
tão infinitamente pequeno quanto a diferença. Contudo, hoje sei que está
errado porque parte de um princípio (1 diferente de 0,999...) errado.

Ralph Costa Teixeira wrote:
> 
> Oi, Gustavo.
> 
> Na minha opiniao, 0,...=1. Sim, eh isso mesmo eh 1. Nao tem
> nada entre esses dois numeros. O primeiro vale 1, o segundo tambem. :)
> 
> Eu mandei uma mensagem uma vez que tentava esclarecer um pouco
> porque as pessoas acham isso surpreendente (nao tanto porque eh 1 -- para
> isso o seu argumento funciona, e hah outros equivalentes como o do
> Daniel). Procure no arquivo da Olimpiada, em Abril de 2000, sob o titulo
> "O Dia que nao acaba".
> 
> Abraco,
> Ralph
> 
> P.S.: O arquivo da Olimpiada que o Nicolau mantem com tanto carinho estah
> em http://www.mat.puc-rio.br/~nicolau/olimp/obm-l.html
> 
> P.S.2: Alias, esta questao tambem jah havia aparecido antes em Maio de
> 1999. Veja tambem
> 
> http://www.mat.puc-rio.br/~nicolau/olimp/obm-rj.1999/threads.html#00143
> sob o titulo "Dizima ou nao-dizima".
> 
> On Sat, 23 Jun 2001, Augusto Morgado wrote:
> 
> > Eu acho estranho que alguns achem estranho 0,999...=1 e não achem
> > estranho 0,333...=1/3.
> > Morgado
> >
> > Gustavo Nunes Martins wrote:
> > >
> > > 8/9 = 0,888...
> > > 1/9 = 0,111...
> > > 0,888... + 0,111... = 0,999...
> > > 8/9 + 1/9 = 9/9 = 1
> > > Entao:
> > > 0,999... = 1
> > >
> > > Nao entendo que 0,999... seja IGUAL a 1. Suponho que seja diferente de
> > > 1. Alguem pode me explicar o que esta coisa significa?
> > >
> > > Atenciosamente,
> > > Gustavo
> >



Re: En: Ligar os Pontos

2001-06-21 Por tôpico Alexandre Tessarollo



 
Hugo Iver Vasconcelos Goncalves wrote:
  Aí vai um desafio
que um colega meu viu e anda quebrando a cabeça do pessoal lah da
minha turma, ligar usando linhas os pontos A, B, C aos pontos D, E, F sem
que essa linhas se cruzem. Isso eh mesmo possível??? A
.  
B .   
C .   D .  
E .   
F . A-D uma linha, A-E uma linha, A-F uma linha,
B-D uma linha, B-E uma linha etc.Será que dava
pra alguém mandar um desenho dessa solução (se eh
que ela existe)???

Caro Hugo,
    esse é um velho problema que não tem
solução. Quando me foi apresentado, dizia que vc tinha três
casas (A, B e C) e três serviços como água, luz e telefone
(D, E e F). A idéia era ligar cada casa aos três serviços
de modo que as ligações não se cruzassem. Ou seja,
exatamente o mesmo enunciado que vc passou, só que com uma historinha.
    A prova de que tal problema não tem solução
só se aprende na faculdade, num ramo da Matemática chamado
Topologia. Não só se prova que no plano euclidiano (o nosso
plano habitual, algo como uma face de um folha de papel) não há
solução, como que o problema pode ser resolvido num toro
(a superfície externa de uma câmara de ar de um pneu de caminhão).
Provavelmente alguém já deve ter te mostrado uma "solução"
que apresentava todas as ligações conforme pedidas, exceto
por uma, que, por exemplo, sai de E e chega em B por "trás" da folha.
    Contudo, ainda não comecei esta parte de topologia
- afinal, ainda estou no terceiro período :-) Agora, se alguém
aqui na lista se dispuser a demonstrar isto...
[]'s
Alexandre Tessarollo




Re: Questões de combinatória/jogos

2001-06-21 Por tôpico Alexandre Tessarollo



Marcelo Rufino de Oliveira wrote:

> Abaixo vão 2 problemas de combinatória/jogos que eu ainda não consegui
> fazer.
> Já mandei estas mesmas duas questões anteriormente para a lista mas
> infelizmente ninguém se manifestou... vamos ver se desta vez alguém pode me
> ajudar.
> Já agradeço, de antemão, aos participantes da lista que tentarem fazer algum
> dos problemas, pois estes não são elementares.
>
> 1) O conjunto {1, 2, ..., 49} é particionado em 3 subconjuntos disjuntos.
> Mostre que ao menos um dos subconjuntos possui três números a, b e c tais
> que a + b = c.
>

Hum, vamos ver...
1a hipótese: Separamos de acordo com o resto na divisão por 3.

Assim, temos o grupo que resta 1, o que resta 2 e o que não resta nada. Neste
último, basta pegar números a=3k, b=3j e c=3(k+j). Naturalmente, k e j são
naturais não-nulos, k é diferente de j e k+j<17. (Isto para que a,b e c estejam
no conjunto original {1,..,49})

Ih, tô vendo que vai dar um certo trabalho e eu tenho aula daqui a dez
minutos... Bem, veja se consegue mostrar o que o problema pede pensando nessas
possibilidades. Talvez tenha uma maneira mais direta, não sei. Vou ver se até
amanhã eu consigo resolver e digitar tudo.

[]'s

Alexandre Tessarollo

PS: Sei que não é a resolução completa, mas de repente ajuda... :-)


>
> 2) Dado um retângulo 1993x1994, dois jogadores (um de cada vez) escreve os
> números 0 ou 1 nas casas. Quando o tabuleiro  está completo seja A o máximo
> valor das somas das 1993 linhas e B o máximo valor das somas das colunas. No
> caso em que A > B o primeiro ganha, no outro caso B ganha. Quem possui uma
> estratégia vencedora?
>
> Falou,
> Marcelo Rufino




Newton (de novo)

2001-06-21 Por tôpico Alexandre Tessarollo

Bueñas!

Mandei umas perguntas recentemente, mas, como todos estavam hã,
empolgados com a OBM, elas (as perguntas) não receberam nenhuma
atenção... snif ;-(

1) Dado um ângulo de vértice A e um ponto E no "interior" do ângulo,
ache B pertencente a um dos lados (do ângulo) de forma que BD seja
mínimo, onde D é a intersecção da reta BE com o outro lado do ângulo.
Vide figura anexa.

2) Dado uma quadrilátero ABCD circunscrito a uma circunferência de
centro O, conforme a figura, prove que a reta que une os pontos médios
das diagonais passa pelo centro O do círculo.

[]'s

Alexandre Tessarollo

PS: Segundo qum me passou, ambas as questões teriam sido formuladas por
Newton.

 Newton.zip


Re: Equacao irracional

2001-06-18 Por tôpico Alexandre Tessarollo


Hum, se o Érico quiser presentear uma matemática (ou alguma pessoa que
goste MUITO de Mat), creio que o Fábio respondeu muito bem. Já se for
para alguém que não simpatiza muito, recomendo uma que ganhei da minha
namorada:

x^2 + (-2amo)x + [(amo)^2 - (te)^2] = 0

Uma equação do segundo grau ax^2 + bx + c = 0. Utilizando o bom e velho
Báskara, temos as raízes
x1 = amo + te
x2 = amo-te

[]'s

Alexandre Tessarollo

PS: naturalmente, a, m, o, t, e são constantes reais.

Fábio Dias wrote:
> 
> -Mensagem original-
> De: [EMAIL PROTECTED] [mailto:[EMAIL PROTECTED]]Em nome de
> Erico Furukawa
> Enviada em: domingo, 17 de junho de 2001 00:44
> Para: [EMAIL PROTECTED]
> Assunto: Equacao irracional
> 
> >Estou precisando de uma ajuda para um desafio:
> >''Elaborar uma equacao irracional cuja V={amor}, e
> >cuja raiz estranha seja {odio}.''
> >Se poderem me ajudar eu ficarei muito agradecido.
> 
> Não está muito bem definido o que o desafio quer dizer com as duas palavras.
> Uma hipótese é que as palavras sejam números em bases suficientemente altas,
> já que depois dos do 0, 1, ..., 9, se passa a utilizar letras.
> 
> Várias escolhas de base são naturais para o problema:
> - base 23 (*todos* os dígitos são representados por letras a..z sem k, w, y)
> - base 26 (*todos* os dígitos são representados por letras a..z com k, w, y)
> - base 33 (além de 0..9, os outros dígitos são representados por letras a..z
> sem k, w, y)
> - base 36 (além de 0..9, os outros dígitos são representados por letras a..z
> com k, w, y)
> 
> Como são duas raízes que estão determinando a equação, é natural que tenha
> dois parâmetros. Vou tomar como equação-modelo sqrt(x + a) = b - x. O lado
> direito da inequação se torna x - b no caso "odio", já que é raiz estranha.
> 
> Os valores de a e b, obtidos no Maple, foram:
> BASE 23: a = 5915129246, b = 95764
> BASE 26: a = 14378381328, b = 146682
> BASE 33: a = 52509302416, b = 649234
> BASE 36: a = 102815878548, b = 864602
> 
> Os valores obtidos não são nada pequenos, mas funcionam.
> 
> []s,
> 
> 
> Fabio Dias ([EMAIL PROTECTED], ICQ# 31136103)
>   RPG em Revista: A sua revista virtual de RPG!
>  --> http://www.rpgemrevista.f2s.com <--



Quadrilátero do Newton

2001-06-12 Por tôpico Alexandre Tessarollo


Aproveitando o ensejo, outra questão do Newton:

"[Prove que,] Dado um quadrilátero ABCD circunscrito a uma
circunferência LAMBDA, a reta que une os pontos médios das diagonais [AC
e BD] passa pelo centro de LAMBDA."

É, dessa vez dica meio difícil fazer uma figurinha em ASCII...

[]'s

Alexandre Tessarollo




Quadrilátero inscrito

2001-06-12 Por tôpico Alexandre Tessarollo


Dado um retângulo ABCD e os pontos P, Q, R e S pertencentes aos
segmentos AB, BC, CD e DA, respectivamente, de forma que DP=AQ=BR=CS= x.
Determine x de forma que a área de PQRS seja mínima.

[]'s

Alexandre Tessarollo




Ângulo do Newton

2001-06-12 Por tôpico Alexandre Tessarollo


Uma questão que, segundo me passaram, foi formulada por Newton:

"Dado um ângulo [agudo(?)] Â e um ponto E [interior ao ângulo Â]
encontre B de forma que [o comprimento] BED seja mínimo."
[adendos meus, para quem não entender a "figura"].

   /
 B
/  \
   / \
 / \
   /   E
 /\
   /__\_
A  D


[]'s

Alexandre Tessarollo




[Fwd: [EMHL] Mir Publishing]

2001-06-07 Por tôpico Alexandre Tessarollo


A resposta que obtive na outra lista..

[]'s

Alexandre Tessarollo

"Paul A. BLAGA" wrote:

> On Wed, 6 Jun 2001, Alexandre Tessarollo wrote:
> The Mir Publishers does not exist anymore, as far as I know, but there is
> another publishing house which continues their work, EDitorial URSS. The
> home page is, as far as I remember, http://urss.ru.
>
> They still have a lot of books from the former MIR.
>
> Paul Blaga
> >
> > Does anybody knows about MIR publuishing? Do they have some kind of
> > home-page, some site where we can find their books or any indication?
> >
> > []'s
> >
> > A. Tessarollo
> >
> > PS: I know this message may be considered as an "off-topic" one. It's
> > just MIR has wonderful books about Maths and particulary about Geometry.
> > :-)
> >
> >
> >
> >
> > Your use of Yahoo! Groups is subject to http://docs.yahoo.com/info/terms/
> >
> >
>
> --
> =
> Dr. Paul A. Blaga
> "Babes-Bolyai" University of Cluj-Napoca
> Faculty of Mathematics and Computer Science,
> Department of Pure Mathematics
> 1, Kogalniceanu Street,
> 3400 Cluj-Napoca, Romania
> e-mail: [EMAIL PROTECTED]
>
>
>
> Your use of Yahoo! Groups is subject to http://docs.yahoo.com/info/terms/




Editora MIR

2001-06-06 Por tôpico Alexandre Tessarollo


Alguém sabe de algum lugar ou site onde possamos encontrar
informações e livros da Editora MIR? Andei procurando pela web, e o mais
próximo que achei é um site que TALVEZ seja o da MIR: www.mirfin.ru.
Digo talvez pq não falo russo e não entendi patavinas do que estava
escrito. Contudo, foi o site indicado pelo Google ao procurar por MIR
publishing

[]'s

Alexandre Tessarollo

PS: Sei que, com vontade, encontra-se em sebos e similares. Mas não é
tão fácil, especialmente se queremos um exemplar em particular. E também
há a desvantagem de não podermos ver todas as opções "disponíveis"...




[Fwd: [EMHL] Inequalities (two questions)]

2001-06-05 Por tôpico Alexandre Tessarollo


Da outra lista, umas questões interessantes... Alguém se candidata a
provar o que a msg diz ser sabido??

[]'s

Alexandre Tessarollo

PS: Sim, é a lista do Conway...

[EMAIL PROTECTED] wrote:

> 1. Let ABC be a triangle, and A'B'C' a triangle inscribed in ABC.
>
> A well-known problem is this:
>
> Prove that the area of A'B'C' is greater or equal of the area of at least
> one of the triangles AB'C', BC'A', CB'A'.
> Prove that the same is true for the perimeters.
>
> Question: Is the same true for the circumradii or inradii of the same
> triangles?
>
> 2. Let ABC be a triangle and A'B'C' the cevian triangle of a point P.
>
> It is known that:
>
> If P = I (ie AA', BB', CC' are the int. angle bisectors)
>
> then 1/AA' + 1/BB' + 1/CC' > 1/BC + 1/CA + 1/AB.
>
> Is the same true for any point P inside ABC?
>
> Antreas
>
>
>
> Your use of Yahoo! Groups is subject to http://docs.yahoo.com/info/terms/




Re: Olimpíada universitária e Criptografia

2001-06-04 Por tôpico Alexandre Tessarollo


Só para ser preciso: o nome do livro que o JP indicou é "Números
Inteiros e Criptografia RSA" do Severino C. Coutinho, editado pela SBM
na "Coleção Computação e Matemática (SCM)" e vendido por R$25,00.

[]'s

Alexandre Tessarollo

PS: Esse e outro (excelentes) livros podem ser adquiridos na sede da
SBM, no Impa, ou em outros lugares. Consulte a lista de endereços e
(principalmente) a de livros na página da SBM, www.sbm.org.br

**
JP
**

Claro, existe o excelente livro do Collier, isto eh: S.C.Coutinho:
Criptografia SRA e Teoria dos Numeros (ou algo
parecido), editado pela SBM.
JP

**
[Davidson Estanislau]
**

   Caros amigos, gostaria da ajuda de vocês:

   Gostaria primeiro de saber se existe algum livro (em
portugues) sobre criptografia;
   Também gostaria que alguém "traçasse" um programa para a
Olimpíada Universitária.

   Obrigado.

   Davidson

**



Re: O que eh isso????!!!!

2001-06-04 Por tôpico Alexandre Tessarollo


Imagine um ponto. Agora arraste este ponto e vc terá um segmento
medindo, hã, "a". Arraste este segmento até obter um quadrado de lado
"a". (Note que começamos com o aidmensional, fomos para o unidimensional
e depois para o bidimensional, sempre "acrescentando" uma dimensão.)
Arraste o quadrado e vc terá um cubo (tridimensional). E, por que não?,
arrastar o cubo e chegar num "cubo quadridimensional"? Como o Leonardo
Motta já disse, "quando se fala em
espacos com mais de 3 dimensoes, referem-se por "hiper" [tanto o espaço
como os objetos nele contidos]", assim, temos hiperplanos, hipercubos,
hiperetc.. :-) Esta é a melhor visualização que EU consigo para explicar
a pergunta.

Outra maneira é com geometria analítica: Se estivermos em R^1 (a reta
real), as possíveis coordenadas feitas com "0" (zeros) e "1" (uns) (em
que há pelo menos um "1") são (1) e (-1). Já em R^2(o plano), temos
(1,0), (-1,0), (0,1), (0,-1). Em R^3 (o espaço como estamos acostumados)
teremos (1,0,0), (-1,0,0), (0,1,0), (0,-1,0), (0,0,1), (0,0,-1). Por
fim, em R^4 (o hiperespaço), teremos (1,0,0,0), (-1,0,0,0), (0,1,0,0),
(0,-1,0,0), (0,0,1,0), (0,0,-1,0), (0,0,0,1), (0,0,0,-1). Note que é
necessário adicionar uma coordenada a cada acréscimo de dimensão e, por
mais estranho que pareça, eu estou o tempo todo com eixos ortogonais
dois a dois. Ou seja, assim como no espaço os eixos x, y e z são
perpendiculares uns aos outros, também no hiperespaço os eixos x, y, z,
w serão ortogonais. (Não, não é difícil imaginar. É IMPOSSÍVEL! Vc
precisaria ter um terceiro olho para ver R^4. Ou, mais genericamente, vc
precisa de n-1 olhos para ver R^n.)

[]'s

Alexandre Tessarollo

PS: De acordo com a minha última observação, em quantas dimensões as
moscas enxergam? :-P

PS2: Vale lembrar que eu trabalhei o tempo todo no espaço euclidiano,
bem comportado, etc, etc, ect. Só não sei qual o termo mais adequado
para designar R^n, com n>3, se hiperespaço ou hiperplano. Tampouco sei
se o quarto eixo ortogonal é comumente chamado de w, ou sequer se recebe
um "nome" especial. Corrijam-me caso tenha cometido algum erro.


> Hugo Iver Vasconcelos Goncalves wrote:
> 
> Alguém poderia explicar o que eh  um "hipercubo tetradimensional" ???



Re: O Jogo "Vida" - Adendo

2001-05-30 Por tôpico Alexandre Tessarollo


Só um detalhe: no arquivo que mandei anexo, TODAS as casas possuem oito
vizinhos.  A figura está num toro. Ou, para quem não conhece toro: as casas da
extrema direita, por exemplo, também possuem como vizinhas as casas da extrema
esquerda. Da mesma forma, as casa de cima e debaixo são vizinhas. Quanto às dos
cantos, basta observar que estão ao mesmo tempo numa lateral e na parte superior
ou inferior. Espero ter sido claro.

[]'s

Alexandre Tessarollo




Re: Problema de Geometria

2001-04-17 Por tôpico Alexandre Tessarollo

Como admirador e estudioso da geometria, só tenho a elogiar sua prova. Realmente
bonita. Meus sinceros parabéns!

[]'s

Alexandre Tessarollo

Edson Ricardo de Andrade Silva wrote:

> Antes de mais nada, vamos a uma breve apresentacao. Sou
> participante desta lista ha uns meses e nao tenho tido muito tempo para
> debater com vcs aqui da lista pois estou condenado a terminar de escrever
> minha tese de mestrado aqui na UFC (Na Area de Computacao Grafica,
> precisamente na area de Modelagem Digital de Terrenos) e o tempo me
> falta... no entanto, acompanho ativamente essa lista como ouvinte.
> Bem, esse problema de geometria me eh particularmente
> interessante. Lembro-me que no tempo do 2 grau, quando eu participava das
> Olimpiadas de Matematica, havia me deparado com esse problema. O ataquei
> com todas as ferramentas que eu dispunha na epoca (geom. plana,
> analitica,transformacoes geometricas...) e nao consegui resolve-lo. Talvez
> a minha decepcao maior foi nao ter encontrado uma solucao atraves de
> geometria plana simples...
> Hoje, porem, quase que num reflexo (apesar de eu estar um pouco
> enferrujado em resolver problemas de olimpiadas), encontrei uma solucao
> bem simples para o problema. Ai vai:
>
> - Considere um novo triangulo B'C'D' como a rotacao de 90 graus do
> triangulo BCD em torno de D e no sentido horario (considerando o pentagono
> ABCDE descrito non sentido anti-horario). Tal rotacao faz o lado C'D'
> coincidir com o lado ED, pois o angulo CDE eh reto e C'D' = CD = ED. Alem
> disso, BD = B'D' e o angulo BDB' eh reto.(***)
>
> - Nao eh dificil observar que o quadrilatero ABEB' eh um
> paralelogramo:
> Observe que EB'= BC = BA. Entao falta provar que EB'// BA. Chame o
> angulo BCD = B'ED = x. Chame a intersecao do prolongamento de EB' com BC
> (ou com o possivel prolongamento de BC) de G. O quadrilatero GCDE eh
> inscritivel, pois B'ED = GCD = x. Como EDC eh reto, temos que ter o angulo
> EGC tambem reto. Logo, os lados EB' e BA sao paralelos, pois fazem os
> mesmo angulos (retos) com BC.
>
> - Agora eh simples. As diagonais B'B e AE do paralelogramo ABEB' se
> cruzam, obviamente, em M, com BM = MB'. Acontece que, como vimos
> anteriormente, o triangulo BDB' eh retangulo em D e isosceles (***), logo
> a altura relativa ao vertice D, ou seja DM, eh igual a metade da
> hipotenusa BB', ou seja BM. E obviamente o angulo DMB eh reto, pois DM
> eh altura.
> CQD.
>
> Eh isso ai gente, espero poder ter ajudado.
>
> PS : Se algum participante da lista tiver interesse na area de Computacao
> Grafica, precisamente nas areas de Geometria Computacional, Modelagem
> Digital de Terrenos, Visualizacao 3D e quiser manter contato, sinta-se
> livre! :)
>
> Abracos,
> Edson Ricardo
>
>  On Fri, 13 Apr 2001, Marcio A. A. Cohen wrote:
>
> >   Como se falou um pouco de complexos aqui, segue abaixo um problema
> > interessante de geometria. Interessante no sentido de ser um problema
> > conhecido, que eu acho bem dificil de se resolver por geometria plana
> > simples, e bem facil de se resolver com auxilio de numeros complexos (e o
> > melhor, eh desses que com geometria analitica convencional continuam
> > dificeis!):
> >
> >   Eh dado um pentagono convexo ABCDE. Sabe-se que AB=BC,  CD=DE, e os angulos
> > internos B e D do pentagono sao de 90 graus. Seja M o ponto medio do lado
> > AE. Demonstre que os segmentos DM e BM sao iguais, e que o triangulo DBM eh
> > retangulo.
> >
> >   Abracos,
> >   Marcio
> >
> > PS: Solucoes simples por geometria plana sao bem vindas, pra desbancar o meu
> > "dificil de se resolver por...". se alguem quiser a solucao por complexos,
> > eh soh lembrar que multiplica um vetor por 90 graus eh multiplicar por
> >
> > PS: Solucoes simples por geometria plana sao bem vindas, pra desbancar o meu
> > "dificil de se resolver por...". se alguem quiser a solucao por complexos,
> > eh soh lembrar que multiplica um vetor por 90 graus eh multiplicar por
> > cis^(90) = i, e desenhar o pentagono no plano.. o resto sao soh poucas
> > linhas de conta.
> >
> >




Re: geometrias & triângulo com mais de 180o ?

2001-04-11 Por tôpico Alexandre Tessarollo



Rogerio Fajardo wrote:

> Por dois pontos passa uma única reta não decorre do axioma das paralelas? Ou
> é um axioma?

Não decorre dos axiomas das paralelas. Pois, se tal axioma diz que "por um
ponto fora da reta, passa uma e somente uma reta paralela à primeira", como
vamos entendê-lo, se não soubermos o que é "reta"?
Mas agora, pensando bem, eu posso ter confundido as coisas (escrever com
pressa não dá certo :-)). Até onde eu me lembre, pontos, retas, planos e algumas
outras coisas são "entes primitivos", istoé, não são definidos, presume-se que
todos concordem sobre o que são.

>
> Aliás, quais são os axiomas de Euclides?

De qualquer forma, para um estudo cuidadoso, com cada axioma, cada teorema, etc,
recomendo o livro que usei em Geometria I. Chama-se "Geometria Euclidiana Plana"
do João Lucas Marques Barbosa. Encontra-se à venda pela SBM, em vários lugares,
inclusive no IMPA.

A pro

>
> >From: "josimat" <[EMAIL PROTECTED]>
> >Reply-To: [EMAIL PROTECTED]
> >To: <[EMAIL PROTECTED]>
> >Subject: Re: geometrias & triângulo com mais de 180o ?
> >Date: Mon, 9 Apr 2001 23:52:26 -0300
> >
> >Essa tal demonstracao errada nao seria de Lagrange, em vez de Legendre?
> >[]s, Josimar

Não sei se Lagrange também errou ou se sequer tentou, mas estou fazendo uma
cadeira aqui na UFRJ na qual estamos estudando os livros do Legendre, e foi nele
que estava o erro.


[]'s

Alexandre Tessarollo

>
> > -Mensagem original-
> > De: Alek <[EMAIL PROTECTED]>
> > Para: [EMAIL PROTECTED] <[EMAIL PROTECTED]>
> > Data: Terça-feira, 10 de Abril de 2001 22:12
> > Assunto: Re: geometrias & triângulo com mais de 180o ?
> >
> >
> > Desculpe, mas faz pouco tempo a professora de algebra linear provou
> >que por dois pontos distintos passa somente uma unica reta(na euclidiana).
> > Portanto ou essa definiçao de axioma esta errada, ou isso de
> >2pontos1reta nao é axioma.
> > ¿¿¿Alguem pode resolver este misterio???
> >
> >
> >
> > At 17:39 09/04/01 -0300, you wrote:
> >
> >
> > Como alguns devem saber, Euclides foi o primeiro a formalizar
> >a geometria e,
> > para tanto, usou alguns axiomas (ou postulados) para provar cada
> >teorema.
> > (Axioma é algo que não pode ser provado e que o bom senso diz ser
> >verdadeiro. Um
> > exemplo de axioma seria o de que "por dois pontos distintos passa
> >uma e somente
> > uma reta")
> >
> > Várias geometrias foram construídas ao longo dos tempos,
> >excluindo um ou
> > outro axioma. Em geral, devido a sua não-obviedade, o primeiro
> >axioma a ser
> > excluído era o das paralelas (dados uma reta r e uma ponto P não
> >pertencente a
> > r, existe um e somente uma reta paralela a r que passa por P).
> >Assim surgiram as
> > geometrias não-euclidianas, com várias aplicações teóricas e
> >algumas práticas.
> > Resumidamente, são classificadas de acordo com a soma dos ângulos
> >internos de um
> > triângulo: maior que 180 ou menor que 180.
> >
> > A geometria esférica (ou da esfera de Rienman) é aquela onde
> >as retas são os
> > círculos máximos, isto é, de centro no centro da esfera e raio até
> >um ponto
> > desta. Com tais retas, pode-se construir um triângulo com três
> >(!!) ângulos
> > retos. Imagine o meridiano de Greenwich, o de 90 graus e o
> >equador. Se
> > necessário, pegue um globo terrestre. É fácil ver que o V
> >postulado (o axioma
> > das paralelas escrito por Euclides) não vale nessa geometria.
> >
> > A geometria do Plano de Poincaré (é essa a geometria
> >elíptica?)  toma a
> > região do plano cartesiano onde y>0 e adota como retas x = k e
> >arcos de
> > circunferências centradas no eixo x e com raios quaisquer. Pode
> >parecer estranho
> > à primeira vista (e realmente é), mas, assim, vc pode construir um
> >triângulo com
> > menos de 180.
> >
> > Já a geometria Euclidiana foi reescrita por alguns, como por
> >exemplo por
> > Legendre. Legendre resolveu adotar outros postulados para
> >construir a mesma
> > geometria de Euclides. Contudo, ao tentar provar o V postulado,
> >ele cometeu um
> > erro de raciocínio que passou indetectado por anos. Isto é, todos
> >sabiam que
> > haviam um erro n

Re: não-euclidiano

2001-04-11 Por tôpico Alexandre Tessarollo



Benjamin Hinrichs wrote:

> Colegas,
>
> tenho uma pergunta que me tem intrigado e não tenho conseguido provar
> com certeza a minha idéia. A pergunta é: quanto vale a área de um
> triângulo esférico (sobre uma esfera, não sei se isso está implícito em
> "esférico"...) de área máxima para uma esfera de raio r. Tenho imaginado
> que é quase a metade da área da esfera, ou seja, 2*Pi*r^2... mas não sei
> se não há algum maior... aliás, não sei se fui muito claro na minha
> exposição de idéias.
>

Hum, vamos ver se eu entendi: Vc imaginou aquele triângulo de 3 ângulos
retos na esfera, de área T = 1/8 da superfície E da esfera e foi movendo um
dos pontos da "base" ao longo do equador até chegar bem perto do outro. Até
aí eu concordo. Mas será que vc não poderia pegar a "reta" que os une e,
como uma alça, puxa-la para baixo? Isto é, fazê-la varrer o hemisfério sul?
Aplicando este racíocionio, acho que se pegarmos três pontos distintos mas
infinitamente próximos, as retas que os unem dois a dois possuem uma parte
"menor" e outra "maior". Assim, tomando os segmentos maiores, poderíamos ter
um triângulo cuja área '"quase" igual a da esfera, isto é, tende a da esfera
e, para um "epsilon" aceitável, T = E. Como eu não estudei geometria
esférica ainda, posso estar falando as maiores asneiras da paróquia.
Agradeço eventuais correções e/ou comentários.

>
> Grande abraço,
>
> Benjamin Hinrichs

[]'s

Alexnadre Tessarollo




Re: Olá também

2001-04-09 Por tôpico Alexandre Tessarollo



Marcelo Souza wrote:

>  Eu mandei umas definições agora...é só dar uma lidaé
> interessante, apesar de naum esclarecer muito
>  >From: "maira.carnaval"
> >Reply-To: [EMAIL PROTECTED]
> >To: [EMAIL PROTECTED]
> >Subject: Olá também
> >Date: Sun, 8 Apr 2001 11:22:03 -0300
> >
> > Antes de tudo, gostaria de que todos me desculpassem
> >pelo incomodo. Eu sei que o tempo de vcs é importante
> >demais para ficarem perdendo com qualquer coisa.

Ninguém perde tempo aqui. As perguntas mais simples são em geral as mais
difíceis. Responder determinadas questões usando apenas matemática de
Ensino Médio pode ser muito difícil, depois que vc se acostuma com as
ferramentas que a faculdade te dá. Não tenha medo e/ou vergonha de fazer
qualquer pergunta aqui na lista, só porque fulano ou sicrano fazem parte
dela. Exatamente porque eles fazem parte da lista é que vc deve
perguntar tudo.

>
> > Mas a questão é que, assim como o Renam, eu estou
> >iniciando agora nessa maravilhosa lista de
> >discussões.

Sejam bem-vindos. Sintam-se à vontade para trazer amigos e conhecidos
que, como nós, apreciem e gostem de matemática.

>
> > Então, eu acho que eu tb deveria me apresentar: meu
> >nome é Maíra, estudo no Franco-Brasileiro, na 3ª série
> >do E.M., e por pura coincidência, o meu caso é
> >exatamente igual ao do nosso amigo Renam. O meu
> >professor de matemática Poncio ( até o nome é muito
> >parecido), que tb é membro dessa lista, foi quem me
> >aconselhou a participar.

Pôncio Mineiro da Silva?!?!?! Mande lembranças a ele, à Moira e ao
Ballet. Ele entenderá ;-)

>
> > Estou adorando!!! E, se possível, gostaria de saber
> >um pouco mais sobre as geometrias euclidiana e não-
> >euclidiana...
> > Muito obrigada pela atenção.
> > Maíra
> >

Espero que estejamos conseguindo te ajudar..

[]'s

Alexandre Tessarollo




Comprimento da Espiral Equiangular

2001-04-09 Por tôpico Alexandre Tessarollo

Bueñas!

Resolvi parametrizar a espiral equiangular sozinho, no braço. Achei
que suas equações seriam

x = Rcos^n(t)cos(nt)
y = Rcos^n(t)sen(nt)

onde R é o raio da circunferência original, t é o ângulo formado
entre OP[n] e OP[n+1].

Todavia, quando fui calcular o comprimento com N de zero a infinito,
cheguei na série
somatório(cos^n(t)/n), n= 1, 2.. infinito.
pretendia achar algo em função de t e depois fazer o limite quando t
->zero, só que ainda não consegui calcular o valor da série. Vale
lembrar que 0
 EspEquiang.zip


Re: geometrias & triângulo com mais de 180o?

2001-04-09 Por tôpico Alexandre Tessarollo


Como alguns devem saber, Euclides foi o primeiro a formalizar a geometria e,
para tanto, usou alguns axiomas (ou postulados) para provar cada teorema.
(Axioma é algo que não pode ser provado e que o bom senso diz ser verdadeiro. Um
exemplo de axioma seria o de que "por dois pontos distintos passa uma e somente
uma reta")

Várias geometrias foram construídas ao longo dos tempos, excluindo um ou
outro axioma. Em geral, devido a sua não-obviedade, o primeiro axioma a ser
excluído era o das paralelas (dados uma reta r e uma ponto P não pertencente a
r, existe um e somente uma reta paralela a r que passa por P). Assim surgiram as
geometrias não-euclidianas, com várias aplicações teóricas e algumas práticas.
Resumidamente, são classificadas de acordo com a soma dos ângulos internos de um
triângulo: maior que 180 ou menor que 180.

A geometria esférica (ou da esfera de Rienman) é aquela onde as retas são os
círculos máximos, isto é, de centro no centro da esfera e raio até um ponto
desta. Com tais retas, pode-se construir um triângulo com três (!!) ângulos
retos. Imagine o meridiano de Greenwich, o de 90 graus e o equador. Se
necessário, pegue um globo terrestre. É fácil ver que o V postulado (o axioma
das paralelas escrito por Euclides) não vale nessa geometria.

A geometria do Plano de Poincaré (é essa a geometria elíptica?)  toma a
região do plano cartesiano onde y>0 e adota como retas x = k e arcos de
circunferências centradas no eixo x e com raios quaisquer. Pode parecer estranho
à primeira vista (e realmente é), mas, assim, vc pode construir um triângulo com
menos de 180.

Já a geometria Euclidiana foi reescrita por alguns, como por exemplo por
Legendre. Legendre resolveu adotar outros postulados para construir a mesma
geometria de Euclides. Contudo, ao tentar provar o V postulado, ele cometeu um
erro de raciocínio que passou indetectado por anos. Isto é, todos sabiam que
haviam um erro na argumentação dele, mas não conseguiam achá-lo. Quem tiver
acesso, vale a pena dar uma olhada nos livros dele.

Espero ter sido de alguma ajuda e/ou esclarecimento e espero também que
alguém me corrija se tiver falado umas besteirinhas... :-)

[]'s

Alexandre Tessarollo

PS: Villard, vc deve (ou pelo menos deveria) estar vendo estas noções (e não o
estudo aprofundado) em Geometria I com a Walcy. Estudo de geometrias
não-euclidianas MESMO, só em Geometria II, que é eletiva. A propósito, bem
vindo, calouro. Abraços do seu veterano.. hehe

Rodrigo Villard Milet wrote:

> Sim ! Se você tiver soma dos ângulos internos igual a 180, com certeza está
> presente o axioma das paralelas !
>  ¡ Villard !
> -Mensagem original-
> De: Rogerio Fajardo <[EMAIL PROTECTED]>
> Para: [EMAIL PROTECTED] <[EMAIL PROTECTED]>
> Data: Segunda-feira, 9 de Abril de 2001 11:40
> Assunto: Re: triângulo com mais de 180o?
>
> >
> >Isso significa que poderíamos substituir o axioma das paralelas pelo
> >axioma: "Existe um triângulo em que a soma dos ângulos é 180°"? Isto é,
> >a existência de um triângulo cuja soma dos ângulos é 180° implica o axioma
> >das paralelas e, consequentemente, que em todos os triângulos a soma dos
> >ângulos é 180°?
> >
> >>From: "Antonio" <[EMAIL PROTECTED]>
> >>Reply-To: [EMAIL PROTECTED]
> >>To: <[EMAIL PROTECTED]>
> >>Subject: Re: triângulo com mais de 180o?
> >>Date: Sun, 8 Apr 2001 18:46:03 -0300
> >>
> >> Até onde eu saiba, em geometrias não euclidianas, a soma dos ângulos
> >>do
> >>triângulo pode ser tanto menor qto maior do que 180 graus.
> >> Mas como esta não é minha especialidade, deixo para os mestres da
> >>lista
> >>comentarem mais o assunto!
> >>
> >>- Original Message -
> >>From: "Rodrigo Villard Milet" <[EMAIL PROTECTED]>
> >>To: <[EMAIL PROTECTED]>
> >>Sent: Sunday, April 08, 2001 1:14 AM
> >>Subject: Re: triângulo com mais de 180o?
> >>
> >>
> >> > A soma dos ângulos internos de um triângulo só é 180 graus na geometria
> >> > euclidiana. Explicanco melhor : Se você verificar que a soma dos
> ângulos
> >> > internos de um triângulo é 180, você só pode estar trabalhando com a
> >> > geometria euclidiana. De fato, num triânguo esférico, a soma dos
> ângulos
> >> > internos do triângulo é > 180 graus. Mas esse triângulo não é definido
> >>na
> >> > geometria plana euclidiana. Note que a prova de que a soma dos angulos
> é
> >>180
> >> > decorre do axioma das paralelas, que só é definido na geo euclidiana.
> >> >  Certamente, se você considerar uma geometria na superfície de uma
> >>esfera,
> >&g

Re: Combinações afins e vetores transportadores no espaço

2000-10-30 Por tôpico Alexandre Tessarollo


Pq? Não entedi.. Veja só:
aA + bB +..+zZ = A + bAB + cAC +..+zAZ
aA + bB +..+zZ = A + b(B-A) + c(C-A) +..+ z(Z-A)
aA + bB +..+zZ = (1 - (b+c+d+..+z))A +bB + cC +..+zZ
aA = (1 - (b+c+d+..+z))A
a = 1 - (b+c+d+..+z)
a + b + c + d +..+ z = 1

Oops!! Não é q fez sentido... Ok, p/não ser um e-mail em vão: Essa
relação vale para Rn? Mesmo p/qq quantidade de pontos?

[]'s

Alexandre Tessarollo

> José Paulo Carneiro wrote:
> 
> Claro que vale!
> JP
> 
> -Mensagem original-
> De: Jorge Peixoto Morais <[EMAIL PROTECTED]>
> Para: [EMAIL PROTECTED] <[EMAIL PROTECTED]>
> Data: Segunda-feira, 30 de Outubro de 2000 20:32
> Assunto: Combinações afins e vetores transportadores no espaço
> 
> No espaço também vale A-B= vetor AB (imagine uma setinha em cima de AB
> apontando para a direita)e Aa +Bb+ Cc+...+Zz= A + bAB + cAC+...+zAZ em
> que as letras minúsculas são coeficientes, as maiúsuclas são pontos,
> duas letras maiúsculas juntas são um vetor e todos os coeficientes
> somam 1?



Re: Curvas de Perseguição

2000-09-28 Por tôpico Alexandre Tessarollo



[EMAIL PROTECTED] wrote:
> 
> Onde está a questão ???
> 
 A questão está enunciada em
<http://www.dmm.im.ufrj.br/projeto/forum/calc.html>

[]'s

Alexandre Tessarollo



Curvas de Perseguição

2000-09-27 Por tôpico Alexandre Tessarollo


Olá a todos!

Trago uma questão que vem me assolando já há algum tempo. A questão
está enunciada em <http://www.dmm.im.ufrj.br/projeto/forum/calc.html>,
com direito até a um *.gif animado p/melhor visualização. Minhas
perguntas são:
- Como se resolve tal questão? 
- É possível generalizar a questão para um polígono regular de n lados?
- Importa se a variação é discreta ou contínua? Pq?

Já fui até perguntar p/a minha professora de Cálculo II e ela disse que
é um problema clássico, resolvido apenas para alguns valores de n. Só
que ela não sabia nada além disso. Alguém pode me ajudar?

[]'s

Alexandre Tessarollo



Re: Prob de triangs antiga...

2000-09-01 Por tôpico Alexandre Tessarollo

O enunciado da questão não sei EXATAMENTE qual era, mas a resolução
abaixo refere-se à "Qual a probabilidade de, escolhidos três pontos
quaisquer do plano (distintos e não-colineares), o triângulo formado por
esses pontos ser acutângulo?" A solução me pareceu clara para o problema
em questão. Qual parte da resolução não está clara?

[]'s

Alexandre Tessarollo

PS: Insisti na resposta do N pq achei que a solução de 1/4 fosse dele,
mas quero ouvir a opinião de outros membros da lista... :-)

"Nicolau C. Saldanha" wrote:
> 
> On Fri, 1 Sep 2000, Alexandre Tessarollo wrote:
> 
> >
> >
> >   Há um bom tempo discutiu-se aqui na lista qual seria a probabilidade
> > de, escolhidos os pontos A, B e C distintos não-colineares, o triângulo
> > ABC ser acutângulo. De acordo com uma resposta (do Nicolau?), seria 1/4.
> > Porém, levei a questào a um grupo de amigos na faculdade e eles
> > discordaram, pq a solução apresentada seria para o caso específico de um
> > triângulo inscrito numa circunferência de raio unitário. Depois de muita
> > discussão, a conclusão foi a seguinte:
> >
> >   "Pegue o maior lado, digamos AB, e trace os arcos de circunferência
> > centrados primeiro em A depois em B, de raio AB. A figura fica
> > semelhante a um olho. O ponto C NECESSARIAMENTE está dentro deste
> > "olho", pois AB é, por hipótese, o maior lado. Trace agora a
> > circunferência que tem AB como diâmetro. Se C estiver na circunferência,
> > ABC é retângulo. Se estiver "dentro" da circunferência, será acutângulo.
> > Se estiver fora da circunferência, será obtusângulo. (Para efeito de
> > cálculo, as áreas de um lado e de outro do segmento AB são iguais.)
> > Fazendo as contas, ou seja, fazendo área do (semi)círculo sobre a
> > (semi)área "externa" do olho (como se fosse a parte branca do olho),
> > chega-se a um número MUITO estranho, porém aceitável e válido para
> > qualquer caso."
> >
> >   O raciocínio está certo? A solução mais antiga também está certa? Se
> > for o caso, qual o erro de qual solução?
> >
> > Aguardando apreciação dos demais (em especial N.),
> >
> > []'s
> >
> > Alexandre Tessarollo
> >
> 
> Um outro membro da lista enviou este problema para a lista com duas respostas,
> uma das quais era 1/4. Minha resposta foi que o problema conforme formulado
> não faz muito sentido. Uma versão que *faz* sentido é se supusermos os três
> pontos em uma circunferência. Existem outras versões que também fazem sentido,
> como por exemplo tomar os três pontos dentro de um disco unitário.
> Estas outras versões devem ter respostas diferentes de 1/4.
> A conclusão dos seus colegas eu não entendi. []s, N.



Prob de triangs antiga...

2000-08-31 Por tôpico Alexandre Tessarollo



Há um bom tempo discutiu-se aqui na lista qual seria a probabilidade
de, escolhidos os pontos A, B e C distintos não-colineares, o triângulo
ABC ser acutângulo. De acordo com uma resposta (do Nicolau?), seria 1/4.
Porém, levei a questào a um grupo de amigos na faculdade e eles
discordaram, pq a solução apresentada seria para o caso específico de um
triângulo inscrito numa circunferência de raio unitário. Depois de muita
discussão, a conclusão foi a seguinte:

"Pegue o maior lado, digamos AB, e trace os arcos de circunferência
centrados primeiro em A depois em B, de raio AB. A figura fica
semelhante a um olho. O ponto C NECESSARIAMENTE está dentro deste
"olho", pois AB é, por hipótese, o maior lado. Trace agora a
circunferência que tem AB como diâmetro. Se C estiver na circunferência,
ABC é retângulo. Se estiver "dentro" da circunferência, será acutângulo.
Se estiver fora da circunferência, será obtusângulo. (Para efeito de
cálculo, as áreas de um lado e de outro do segmento AB são iguais.)
Fazendo as contas, ou seja, fazendo área do (semi)círculo sobre a
(semi)área "externa" do olho (como se fosse a parte branca do olho),
chega-se a um número MUITO estranho, porém aceitável e válido para
qualquer caso."

O raciocínio está certo? A solução mais antiga também está certa? Se
for o caso, qual o erro de qual solução?

Aguardando apreciação dos demais (em especial N.),

[]'s

Alexandre Tessarollo



Colégio Naval

2000-08-31 Por tôpico Alexandre Tessarollo

Olá a todos!

A primeira questão do Colégio Naval/2000.

"Seja ABCD um quadrilátero qualquer onde os lados opostos NÃO são
paralelos. Se as medidas dos lados opostos AB e DC são, respectivamente,
iguais a 12 e 16, um valor possível para o segmento de extremos M (ponto
médio do lado AD) e N (ponto médio do lado BC) é:

a) 12,5
b) 14
c) 14,5
d) 16
e) 17"


Sei que 12


Re: sugestão

2000-08-15 Por tôpico Alexandre Tessarollo



Eu pesssoalmente acho que não é necessário. De fato são comuns
discussões que por vezes nem um aluno iniciando o segundo grau sabe a
matemática necessária para enteder a questão. Porém, se tomramos uma
Eureka, por exemplo, vamos encontrar discussões tanto para iniciantes
como para nível avançado. Discussões além do que podemos entender nos
forçam a correr atrás, procurar saber mais. Ou pura e simplesmente
ignorá-las e apertar a tecla delete. Não é porque todos não entendem
determinada discussão que ela deve ser "discriminada" e jogada para
outra lista Assim como não se escreve uma Eureka para iniciantes, outra
para nível intermediário e uma terceira para nível avançado, não vejo
necessidade de montar uma lista para cada nível. Até porque, uma
pergunta feita por um aluno de segundo grau obtendo uma resposta para
universitários força o aluno a tentar expandir seus conhecimentos como
aquele todo o grupo a pensar numa resposta mais simples, que, por vezes,
são as mais difíceis. Fica aqui minha *opinião* *pessoal*.

[]'s,

Alexandre Tessarollo



Re: Pergunta solta

2000-07-30 Por tôpico Alexandre Tessarollo



Respondendo ao Marcelo: "Maple" é um programa de matemática que faz
MUITA coisa, entre gráficos (todos os tipos), integrais, derivadas, etc.

O único lugar que eu conheço e também onde comprei o meu, foi na UFRJ,
no Instituto de Matemática, no CT, por R$20. Só não sei se a venda é só
para alunos e professores da UFRJ ou se é aberta. Se for, não sei se é o
mesmo preço. O único lugar além do IM que eu acho que possa ter é no
IMPA.

Alexandre Tessarollo



Re: Questão das Olimpíadas

2000-07-27 Por tôpico Alexandre Tessarollo

Seja bem-vindo!

Bem, mãos à obra: 7/10 José Alvino wrote:
> 
> Olá pessoal!
> Sou novato aqui na lista e gostaria que alguém me ajudasse numa
> questão ou me informasse onde posso encontrar sua resolução. É a
> questão 10 da 1a fase junior de 97:
> 
> 
> Se p e q são inteiros positivos tais que 7/10 < p/q < 11/15 , o menor
> valor que q pode ter é:
> 
> A) 6 B) 7 C) 25 D) 30 E) 60
> 
>  Agradeço antecipadamente.
> 
>



Re: Combinatoria

2000-07-19 Por tôpico Alexandre Tessarollo

Essa é uma questão de permutação circular. Fiz de duas maneiras.

Primeira maneira:

Vamos primeiro permutar todas as bolas como se estivessem uma ao lado
da outra numa prateleira. Para quem já estudou permutação c/repetição, é
fácil ver que existem N=54!/(6!8!16!24!) arrumações possíveis. Agora
"fechemos" o círculo, isto é, juntemos uma ponta da prateleira à
outra(como se a prateleira fosse maleável). Ao fazermos isto, vemos q a
arrumação q põe todas as bolas juntas de acordo com a cor, isto é,
BBAzAz..AzVV...VAmAm...Am é equivalente à
BAzAz..AzVV...VAmAm...AmB que é equivalente a várias outras.
Precisamente 54 arrumações equivalentes. Basta ver que o "ponto de
corte" da arrumação acima poderia ter sido em qualquer um dos 53 espaços
entre as bolas bem como aonde admitimos ter sido, no fim da nossa
prateleira.)
Logo, o verdadeiro número de arrumações é N/54.

Segunda maneira:

Escolha uma bola qualquer, digamos branca. Coloque-a em qualquer
posição, pois todas são equivalentes inicialemente. Agora, para colocar
a segunda bola branca, temos o lugar simétrico ao da primeira e mais
52/2=26 lugares (na verdade seriam 52 lugares, só que são simétricos
dois a dois. Logo...). Ou seja, já temos 27 possibilidades. Já podemos
perceber também que dessa maneira teremos vários casos e não chegaremos
ao resultado tão cedo.
Assim, na hora de colocar as bolas seguintes, nós "abrimos" o círculo.
Isto é, assumimos que a 1a bola colocada representa a 1a posição.
Resolvendo essa permutação normal, temos M=53!/(5!8!16!24!). Vale
lembrar que a nossa primeira bola branca NÃO é diferente das outras, ou
seja, existem 6 bolas brancas q podem ser esta primeira. Portanto, o
verdadeiro número de arrumações é M/6.

E, como podemos ver, M/6=N/54. Ou seja, ambos os raciocínios chegam a
mesma resposta e ambos estão, a meu ver, corretos.

Aguardo apreciação de todos.
Um abraço,
Alexandre Tessarollo

Ecass Dodebel wrote:
> 
> "De quantas maneiras distintas podemos dispor ao longo de um
> circulo, suposto fixo, 6 bolas brancas, 8 bolas azuis, 16
> bolas verdes, 24 bolas amarelas?"
> 
> O círculo fica fixo em nossa frente, mas as bolas ficam livres para serem
> rotacionadas como em uma catraca de bicicleta (acho que vocês entendem).
> 
> Obrigado!
> 
> Eduardo Casagrande Stabel.
> 
> Get Your Private, Free E-mail from MSN Hotmail at http://www.hotmail.com



Re: Poligono e Poligonal

2000-07-12 Por tôpico Alexandre Tessarollo

Assim como circuferência é a linha (LG dos pontos etc), poligonal também
é a linha (união dos segmentos...). Da mesma forma, polígono é a região
(intersecção finita de todos os semi-planos determinados pelas retas que
contêm os segmento da poligonal). Certo?

Bruno Woltzenlogel Paleo wrote:
> 
> > O mesmo vale para polígono e poligonal.
> 
> Kual a definição de polígono e poligonal?



Re: circunferência x círculo

2000-07-12 Por tôpico Alexandre Tessarollo

Caro Filho,

No segundo grau aprendemos que circuferência é a linha (que une os
pontos equidistantes etc) e círculo seria a linha + a região interna.
Porém, na Faculdade eu levantei essa questão com minha professora de
Geometria I e ela me disse que para geômetras diferenciais (como ela)
circuferência e círculo seriam apenas a linha, e disco seria a linha + a
região interna. Portanto, *eu* *acho* que devemos variar os termos de
acordo com a, hã, "platéia". Todavia uma uniformização dos termos
facilitaria e muito a vida de todos, alunos, professores e pessoas
envolvidas com geometria em geral.


> Filho wrote:
> 
> É muito comum se ver em livros e provas de vestibulares, perguntas do
> tipo: Determine a área da circunferência.ERRADO OU OK ? Não seria
> melhor: Determinar a área da região limitada pela circunferência ou do
> disco ou mesmo do círculo. Ou será correto se referir a uma
> circunferência como sendo um círculo? A circunferência não é um
> subconjunto do círculo? O círculo não pode ser visto como a reunião de
> várias circunferências concêntricas? Devemos fazer vista grossa?
> Gostaria de ter a opinião de vocês.



Re: Problema(novamente)

2000-04-09 Por tôpico Alexandre Tessarollo



Eduardo Casagrande Stabel wrote:
> 
> +/v8APAAh-DOCTYPE HTML PUBLIC +ACI--//W3C//DTD HTML 4.0
> Transitional//EN+ACIAPg- +ADw-HTML+AD4APA-HEAD+AD4- +ADw-META
> content+AD0AIg-text/html+ADs- charset+AD0-utf-7+ACI-
> http-equiv+AD0-Content-Type+AD4- +ADw-META content+AD0AIg-MSHTML
> 5.00.2314.1000+ACI- name+AD0-GENERATOR+AD4-
> +ADw-STYLE+AD4APA-/STYLE+AD4- +ADw-/HEAD+AD4- +ADw-BODY
> bgColor+AD0AIw-ff+AD4- +ADw-DIV+AD4APA-FONT size+AD0-2+AD4-Estava
> pensando na funcao:+ADw-BR+AD4APA-BR+AD4AJg-nbsp+ADs-x(a , n) +AD0-
> 1+AF4-a +- 2+AF4-a +- ... +- n+AF4-a+ADw-BR+AD4APA-BR+AD4-E tentava
> descobrir qual o resto da divisao de x(a , n) por n de
> forma+ADw-BR+AD4-explicita em termos do a e do n, tem jeito facil para
> fazer isso?+ADw-BR+AD4APA-BR+AD4APA-BR+AD4APA-BR+AD4-PS.
> escolhendo-se+ACY-nbsp+ADsAJg-nbsp+ADs- y(a , n , k) +AD0- k+AF4-a +-
> (k+-1)+AF4-a +- ... +- (k+- n -1)+AF4-a+ACY-nbsp+ADs- ,
> e+ADw-BR+AD4-claramente temos:+ADw-BR+AD4APA-BR+AD4AJg-nbsp+ADs-y(a ,
> n , 1)+AD0-x(a , n)+ADw-BR+AD4APA-BR+AD4-Eh facil de ver que temos y(a
> , n , k) +AD0- x(a , n) (mod n) para qualquer
> k.+ADw-BR+AD4APA-BR+AD4-PS2. a ideia eh tentar resolver um dos
> problemas que ja veio para a lista+ADw-BR+AD4-(com o a+AD0-2), ou algo
> 
>assim.+ADw-BR+AD4APA-BR+AD4APA-BR+AD4APA-/FONT+AD4APA-/DIV+AD4APA-/BODY+AD4APA-/HTML+AD4-


Ok, já foi traduzido do grego para o chinês. Agora será que da para
traduzir para uma língua que ao menos utilize o nosso alfabeto? :-)

Alexandre Tessarollo

PS: Só para visualizar uma figura que eu particularmente achei
interessante:

"Uma amostra de material radioativo foi prensada na forma de um cubo de
aresta a. Deseja-se envolver o cubo com material isolante de forma que
cada ponto da superfície do cubo esteja a uma distância d. Qual o volume
total de material isolante utilizado?" 

Note que a figura fica mais, hum, interessante p/ a=d.